Head & Neck Tumors/Parotid Flashcards

1
Q

An 80-year-old man presents with floor of mouth cancer on the right side. In addition to the 1-cm area of ulceration, there is a hard, warm, nonmobile, painful mass in the submandibular area. This mass most likely originated from which of the following types of tissue?

A) Bone
B) Fat
C) Muscle
D) Nerve
E) Salivary

A

The correct response is Option E.

The submandibular gland duct, also known as the Wharton duct, ends in the area of the floor of the mouth and is typically blocked when cancer invades this area. As a result, there is backup of salivary content, the gland enlarges, becomes firm and painful, and can even become infected. This event has no prognostic implications. However, an enlarged node, such as the perifacial nodes, can mimic this finding and can upstage a cancer patient. If a bimanual examination of the gland cannot be performed because of the patient’s pain, a CT scan is easily confirmatory. In general, lymph nodes are mobile, with the rare exception of external bony erosion. Nonetheless, a complete head and neck physical examination can be key to interpreting the findings described.

Bony expansion from a floor-of-mouth cancer is possible, but will not usually present as a painful mass. In general, fat is not involved. Nerve involvement can also occur, and perineural invasion in the lingual nerve can cause its enlargement, but not with this presentation. A sublingual infection or hematoma (e.g., Ludwig angina) can present with what is typically a swollen bilateral submandibular mass, as the infection/abscess extends below the mylohyoid muscle. This process can cause immediate airway compromise and many recommend an emergent tracheostomy. This pathology is almost always related to bacterial infections of the mouth, most commonly related to dental caries.

2018

How well did you know this?
1
Not at all
2
3
4
5
Perfectly
2
Q

A 59-year-old woman who has worked in rubber manufacturing for 35 years is referred by her primary care physician because of a firm, nontender, 1-cm mass with ulceration on the hard palate between the central incisors and the incisive foramen. This suspected minor salivary gland tumor is most likely to drain into which of the following nodal basins?

A) Submandibular nodes
B) Parathyroid nodes
C) Parotid nodes
D) Posterior triangle nodes
E) Occipital nodes

A

The correct response is Option A.

The area of the lips, gums, teeth, tongue, and anterior hard palate will drain to the submental and submandibular region (Level I).

Levels IA (submental) and IB (submandibular) are separated by the anterior digastric muscle.

Group II drains the naso-/oro-/hypopharynx and parotid and is called the upper jugular group.

Group III ( middle jugular group) drains the naso-/oro-/hypopharynx and larynx.

The lower jugular group (level IV) drains the larynx, cervical esophagus, and hypopharynx.

Group V is the posterior triangle group and drains the naso- and oropharynx.

Group VI is the anterior central group below the hyoid and above the sternal notch. The group drains the thyroid, parathyroid, cervical esophagus, and larynx. More posteriorly the hard palate and soft palate will drain to the retropharyngeal space and deep cervical nodes.

2018

How well did you know this?
1
Not at all
2
3
4
5
Perfectly
3
Q

A 50-year-old woman has a Mohs resection defect starting 15 mm below her lower lip vermilion border and extending 3 cm caudally; the width of the defect is 4.5 cm. Exposed bony mandible is noted in the depth of the wound. A photograph is shown. The simplest and most aesthetically pleasing reconstruction is likely to be based on which of the following arterial branches?

A) Facial
B) Radial
C) Superficial temporal
D) Thoracoacromial
E) Transverse cervical

A

The correct response is Option A.

One of the branches of the facial artery is the submental artery. This vessel provides flow to the so-called submental flap, which can be made myofascial or musculocutaneous as it includes the platysma muscle. The flap is almost adjacent to the defect and provides similar skin in color and texture. The donor site is typically closed primarily after undermining the lower neck.

The thoracoacromial artery provides flow to the pectoralis flap. Although the pectoralis myocutaneous flap could be used here, it would be bulky, would have to exclude breast tissue, would not have a similar color match, and would create a tether from the chest to the chin. The superficial temporal artery can allow for a large laterally based forehead flap; in the era prior to free tissue transfer, this flap was used for even intraoral reconstruction. In theory, such a flap could reach the mental area, but the dissection would be very tedious and the donor site alterations significant. The transverse cervical artery can provide a flap of thin supraclavicular skin that, as a free flap, could be used in this area but certainly would not be the simplest option. Likewise the radial forearm free flap based on the radial vessels would not be a good color match and would be a lengthy, complex surgery.

2018

How well did you know this?
1
Not at all
2
3
4
5
Perfectly
4
Q

A 45-year-old man with mucoepidermoid carcinoma of the left parotid gland undergoes parotidectomy. Which of the following complications is most likely in this patient?

A) Gustatory sweating
B) Hyperacusis
C) Loss of taste
D) Tardive dyskinesia
E) Tearing while eating

A

The correct response is Option A.

Parotidectomy is a surgical procedure performed for benign and malignant tumors of the parotid gland and, uncommonly, for hypersalivation. Specific risks for parotidectomy include facial nerve injury, sialocele formation, and gustatory sweating.

Gustatory sweating (or Frey syndrome) results when there is aberrant innervation of cutaneous sweat glands in the cheek skin overlying the parotid bed resulting in sweating and flushing during eating.

Loss of taste to the anterior two thirds of the tongue results from injury to the chorda tympani, which arises from the facial nerve as it crosses the tympanic cavity and joins with the lingual nerve, not from injury to the facial nerve within the parotid gland.

The stapedius is a small muscle in the middle ear that restricts movement of the stapes bone in response to loud noises. Hyperacusis may occur when the facial nerve is interrupted intracranially, before it exits the stylomastoid foramen and becomes extracranial.

Tearing while eating (crocodile tear syndrome or Bogorad syndrome) is an uncommon consequence of nerve regeneration subsequent to Bell’s palsy in which efferent fibers from the superior salivary nucleus become improperly connected to nerve axons projecting to the lacrimal glands causing tears while smelling foods or eating. Tardive dyskinesia is a disorder resulting in involuntary repetitive movements including grimacing, tongue and lip movements, and excessive blinking that usually occur as side effects of antipsychotic drugs. It is unrelated to facial nerve dysfunction.

2018

How well did you know this?
1
Not at all
2
3
4
5
Perfectly
5
Q

A 32-year-old man is evaluated because of painless swelling of the right cheek 2 months after being assaulted with a knife and sustaining a laceration across the mid cheek. The laceration was sutured by the emergency department physician assistant. Which of the following is the most appropriate management at this time?

A) Drainage of the collection and administration of antisialagogues
B) Open debridement with skin grafting
C) Surgical exploration with reanastomosis of the injured duct
D) Total parotidectomy with facial nerve preservation
E) Ultrasound-guided needle aspiration only

A

The correct response is Option A.

Any penetrating injury on a line from the tragus to the mid upper lip may injure the parotid duct or gland. The parotid system can be divided into 3 regions:

Region A: gland
Region B: duct superficial to the masseter muscle
Region C: duct from the masseter to where it enters the mouth opposite the 2nd maxillary molar

Additionally, the buccal branch of the facial nerve often runs with the duct. It can cross the superficial layer of the masseter after leaving the parotid gland. Injury manifests as weakness of the upper lip on animation.

Injury not repaired acutely can manifest later as salivary fistulae or sialoceles. Most can be managed conservatively with drainage, pressure, and antisialagogues. Trying to find and repair the duct late after the injury is difficult and may not lead to resolution of the symptoms. This is usually reserved for early injury and repair. If these fail, then a superficial parotidectomy can be considered for recalcitrant salivary collections.

2018

How well did you know this?
1
Not at all
2
3
4
5
Perfectly
6
Q

Which of the following viruses is implicated in the pathogenesis of nasopharyngeal carcinoma?

A) Epstein-Barr virus
B) Hepatitis C virus
C) Human herpesvirus 8
D) Human immunodeficiency virus
E) Human papillomavirus

A

The correct response is Option A.

All phases of the Epstein-Barr virus life cycle are associated with human disease. In immunocompromised individuals, infected cells increase in number, and eventually B-cell growth control pathways are activated, inducing transformation and leading to malignancies such as nasopharyngeal carcinoma, Burkitt lymphoma, post-transplant lymphomas, and gastric carcinomas. Human papillomavirus (HPV) is increasingly recognized as a pathogenic risk factor for oropharyngeal cancer development. Accumulating molecular and epidemiological data now show that high-risk types of HPV are responsible for a subset of oropharyngeal cancer. Oral verrucous and squamous cell carcinomas have been reported in patients infected with hepatitis C virus, and the infection has been found to be more prevalent in patients with oral lichen planus. Infection with HIV is not known to be directly pathogenic in malignant transformation, but rather it increases the susceptibility to opportunistic infections and viral-promoted cancers. Human herpesvirus 8, also known as Kaposi sarcoma–associated herpesvirus, has been found in nearly all tumors in patients with Kaposi sarcoma.

2018

How well did you know this?
1
Not at all
2
3
4
5
Perfectly
7
Q

A 21-year-old woman is evaluated for a painless enlargement along the left lower jaw. Panoramic x-ray study (Panorex) is shown. Which of the following is the most likely diagnosis?

A) Cherubism
B) Dentigerous cyst
C) Desmoid tumor
D) Giant cell tumor
E) Tuberous sclerosis

A

The correct response is Option B.

The findings on the panoramic x-ray study (Panorex) are most consistent with a dentigerous cyst, also referred to as a follicular cyst. These lesions are not tumors, but cysts that originate from the separation of the dental follicle from the crown of an unerupted tooth. Thus, dentigerous cysts arise during development and are odontogenic (from the tooth or its precursors) in origin. The cysts are lined by specialized epithelium that is instrumental in the formation of tooth enamel. This tissue usually atrophies and becomes part of the gingiva after the enamel is formed; a cyst forms when fluid accumulates between the atrophied or reduced enamel epithelium and the crown of an unerupted tooth. These cysts are most common around the mandibular third molar (wisdom tooth) and are seen most often in teenagers/young adults. They present as a painless bone expansion and appear on x-ray study as a well-circumscribed, unilocular radiolucency, often with a sclerotic rim. The relationship between the cyst and the tooth varies.

Giant cell tumor, desmoid tumor, and osseous tumors associated with tuberous sclerosis are solid, not cystic lesions. The former two lesions tend to be more erosive on x-ray study and have a largely solid composition. Bony tumors arising in the context of tuberous sclerosis are uncommon, but tend to be solid. Cherubism is a rare autosomal dominant disorder that begins in childhood and in which mandibular and maxillary bone is replaced by fibrous tissue and cysts. It reportedly improves over time but can be disfiguring.

2018

How well did you know this?
1
Not at all
2
3
4
5
Perfectly
8
Q

A 60-year-old woman undergoes left hemiglossectomy for squamous cell carcinoma of the oral tongue. Reconstruction with which of the following is most likely to achieve the best functional outcome?

A) Pectoralis major myocutaneous pedicled flap
B) Radial forearm fasciocutaneous free flap
C) Rectus abdominis myocutaneous free flap
D) Split-thickness skin graft
E) Temporalis muscle pedicled flap

A

The correct response is Option B.

Reconstructive strategies for partial and hemiglossectomy reconstruction are different than for subtotal and total glossectomy reconstruction. For partial and hemiglossectomies, the ideal reconstruction preserves the mobility of the remaining tongue, maintaining its ability to manipulate foods, articulate words, and sweep the oral cavity clean. Such defects are best reconstructed with a thin, pliable flap that resists contraction during the healing process. Of the choices listed, the radial forearm fasciocutaneous free flap best fits those requirements.

The pectoralis major myocutaneous pedicled flap is usually too thick to permit ideal movement and can have problems with reach and tethering to the chest as the proximal flap and pedicle contact during healing. The temporalis muscle pedicled flap is unlikely to reach the entire defect and the muscle is likely to contract and stiffen with time, limiting tongue movement. The rectus abdominis myocutaneous free flap is usually too thick to permit unrestricted movement of the remaining hemitongue and is better suited to subtotal and total glossectomy reconstruction, which is best effected with a bulky flap that diverts food and liquids laterally into the pharynx rather than into the larynx. A split-thickness skin graft, while thin, usually contracts substantially, limiting tongue movement. Also, if there is communication into the neck following tongue resection with neck dissection, the skin graft would be at high risk for resulting in a fistula.

2018

How well did you know this?
1
Not at all
2
3
4
5
Perfectly
9
Q

A 65-year-old man undergoes microvascular reconstruction with a radial forearm flap following total laryngectomy and partial pharyngectomy for a T4 N3 M0 laryngopharyngeal squamous cell carcinoma. Postoperatively, the patient reports perioral numbness and tingling. Which of the following electrolyte abnormalities is most likely in this patient?

A) Hypercalcemia
B) Hyperkalemia
C) Hypernatremia
D) Hypocalcemia
E) Hyponatremia

A

The correct response is Option D.

Patients who undergo laryngopharyngectomy often have the thyroid gland removed as part of the operation. Sometimes, the parathyroid glands are removed accidentally or the blood supply is transiently interrupted, causing poor function. The parathyroid glands regulate calcium metabolism, and injury or removal of these glands manifests first as perioral tingling and numbness before more severe symptoms such as tetani.

Hypernatremia is incorrect because, when symptomatic, it causes nausea, vomiting, and confusion. Hyponatremia is incorrect because, when symptomatic, it causes nausea, vomiting, lethargy, and confusion. Hypocalcemia is the correct answer because it is a concern for any patient who has this operation and reconstruction. Perioral tingling and numbness is an early sign of problems. Hypercalcemia is incorrect because, when symptomatic, it causes gastrointestinal issues such as vomiting, diarrhea, constipation, and skeletal muscle pain. Hyperkalemia is most likely to cause EKG changes and metabolism is not associated with parathyroid function.

2018

How well did you know this?
1
Not at all
2
3
4
5
Perfectly
10
Q

A 37-year-old man comes to the office for evaluation of whitish papules of the oral mucosa 3 weeks after undergoing successful face transplantation encompassing the maxilla, palate, oral mucosa, and overlying skin. Examination shows that the patient is afebrile and has no leukocytosis. No changes have been made to the initial acute postoperative immunomodulation therapy. Which of the following is the most appropriate initial management of this patient?

A) Apply topical nystatin
B) Biopsy for allograft rejection
C) Increase the dose of tacrolimus
D) Initiate oral amoxicillin therapy
E) Reexamine the patient in 4 weeks

A

The correct response is Option A.

Post-transplant infections can be divided into three phases: early postoperative infections (0 to 2 months), intermediate (2 to 9 months), and late (greater than 9 months). Because of the variation in tissue type and anatomy between solid organ transplantation and composite tissue allograft transplantation, there are differences between types of infection. In all transplant patients, cytomegalovirus (CMV) infection/reactivation is the most common infectious complication. As a result, the protocol for composite tissue allograft recipients includes CMV prophylaxis.

In this particular scenario, facial transplantation is unique in that there are oral and sinus cavities with colonization with native flora. Candida is the most ubiquitous and most common flora. Treatment is topical nystatin and clotrimazole.

Although there is constant wariness for rejection, this does not occur commonly in the immediate postoperative period. Biopsy would be warranted if there is no improvement with topical antifungals.

As there is no indication of acute rejection, increasing tacrolimus is not an appropriate initial treatment.

Amoxicillin is a bactericidal antibiotic and is not an appropriate treatment for this fungal infection.

2018

How well did you know this?
1
Not at all
2
3
4
5
Perfectly
11
Q

A 65-year-old man with Stage II squamous cell cancer at the base of the tongue comes to the office. He has a 30-year history of smoking. Which of the following type of neck dissection is most appropriate for this patient?

A) Modified: levels I to V nodes
B) Radical: levels I to V nodes, CN XI, internal jugular vein, sternocleidomastoid muscle removed
C) Selective (anterior compartment): VI
D) Selective (lateral neck): levels II to IV
E) Selective (supraomohyoid): levels I to III

A

The correct response is Option E.

Selective removal of nodes in levels I to III is the procedure of choice for N0 and N1 cancers in the oral cavity. Lateral and anterior compartment dissections miss Level I, which is the closest lymphatic level to a base of tongue cancer. Modified neck dissections are indicated for clinically palpable metastatic disease. Radical neck dissections are indicated for involvement of the nerve, vein, or muscle.

2018

How well did you know this?
1
Not at all
2
3
4
5
Perfectly
12
Q

A 53-year-old man is undergoing revision of mandibular reconstruction. An iliac crest osteocutaneous free flap is planned. Which of the following is the main advantage of this flap compared with a free fibula osteocutaneous flap?

A) Bone length
B) Minimal donor site morbidity
C) Pedicle length
D) Reliability with multiple osteotomies
E) Vertical height

A

The correct response is Option E.

There are many choices for bone grafts, both vascularized and nonvascularized. In this clinical scenario, there is no question that a vascularized bone graft is indicated, given the irradiated field, anterior location, and >6-cm defect.

The choice of vascularized bone grafts include the free fibula, free iliac crest, free scapula, and free radius bone grafts. All have their pros and cons individually, although collectively, vascularized bone free flaps provide 40% more strength, 56% more stiffness, higher complete arthrodesis rate, and superior functional outcomes.

The free fibula flap would be the most common choice for this situation given its long pedicle (6 to 10 cm), the large amount of usable bone (22 to 24 cm), minimal donor site morbidity, ability to accept dental implants, and reliable skin flap (skin island survival rates approaching 100% due to increased anatomical understanding and improvements in harvest techniques). However, its disadvantage is that it doesn’t have the vertical height of the free iliac crest.

2017

How well did you know this?
1
Not at all
2
3
4
5
Perfectly
13
Q

A 57-year-old man undergoes superficial parotidectomy. Facial nerve neuropraxia results in gustatory sweating and which of the following additional symptoms?

A) Anosmia
B) Base of tongue dysgeusia
C) Hyperlacrimation
D) Migraine headache
E) Synkinesis

A

The correct response is Option C.

Hyperlacrimation, or Bogorad syndrome, is a known complication after Bell palsy or other injury and insults to the facial nerve. Similar to Frey syndrome, the predominant theory for this form of gustatory hyperlacrimation is due to aberrant facial nerve regeneration. Epiphora in general can also occur due to poor “pumping mechanisms” in the eyelids as well as prolonged ectropion and conjunctival show after facial nerve injury. However, hyperlacrimation during gustatory activity is a specific and definable pathology. Treatment for this syndrome includes subtotal lacrimal gland resection, botulinum toxin type A, and various forms of enlarging the lacrimal tract.

Synkinesis is a common event after facial nerve regeneration, when the nerve improperly fires and there is lack of typical mimetic muscle coordination. The anterior portion of the tongue taste buds are innervated by facial nerve fibers from the chorda tympani to the lingual nerve, but the base of the tongue is innervated by cranial nerves IX and X. Anosmia is loss of smell that occurs through cranial nerve I injury or obstruction and can lead to taste disturbances. Migraine headaches can be associated with a variety of syndromes and need to be differentiated from other forms of headaches. Ramsay Hunt syndrome can lead to facial nerve dysfunction and facial pain, but this pain is not associated with facial nerve regeneration or migraine headaches.

2017

How well did you know this?
1
Not at all
2
3
4
5
Perfectly
14
Q

An 18-year-old man comes to the office for evaluation because of swelling of his chin. A panoramic x-ray study (Panorex) is shown. Which of the following types of cyst is the most likely diagnosis?

A) Dentigerous
B) Gingival
C) Periapical
D) Primordial
E) Residual

A

The correct response is Option A.

This radiograph is most consistent with a dentigerous cyst.

Dentigerous cysts are the second most common and develop in the dental follicle of an unerupted tooth. On radiograph there is usually a lucency attached at an acute angle to the tooth. The mandibular and maxillary third molars are the most commonly affected.

Odontogenic cysts are epithelial lined cysts that are defined by location and histologic characteristics.

Periapical cysts are the most common and usually form from necrotic pulp after a tooth infection. They usually present as a radiologic lucency at the apex of the tooth.

A gingival cyst is a superficial cyst in the gingiva. A primordial cyst develops instead of a tooth. This is a rare cyst.

A residual cyst may result from a retained periapical cyst after teeth have been removed.

2017

How well did you know this?
1
Not at all
2
3
4
5
Perfectly
15
Q

A 35-year-old woman undergoes surgical resection of a left parotid gland malignancy. The facial nerve was resected with the tumor, leaving a 5- to 7-cm gap between the proximal nerve stump and the distal nerve branches. Which of the following is the most appropriate treatment?

A) Cable nerve grafting
B) Cross-facial nerve grafting
C) Hypoglossal nerve to facial nerve transfer
D) Innervated gracilis muscle free flap reconstruction
E) Nerve repair with a conduit

A

The correct response is Option A.

When a facial nerve has been divided or resected, the best outcomes for regaining function are usually obtained from direct repair, or cable nerve grafting when too great a distance for direct repair separates the nerve ends.

While autologous nerve grafts from “expendable” donor nerves, such as the great auricular nerve or sural nerve, have long been the gold standard, nerve repair using biologic or synthetic nerve conduits has also produced reasonable results, in some series equivalent to cable nerve grafts. Conduit nerve repair has the advantage of having no donor site morbidity. However, the length of the gap between the proximal and distal cut nerve ends is usually limited to less than 3 cm for the best chances of nerve recovery.

When direct repair or cable nerve grafting is not possible—for example, when the nerve has been resected very proximally up to the intracranial portion of the nerve—cross-facial nerve grafting between redundant branches of the normal contralateral nerve and the distal facial nerve stumps of the paralyzed side can produce reasonable results with spontaneous symmetrical facial movement. Performing a nerve transfer from a donor nerve, such as the masseteric (V), spinal accessory (XI), or hypoglossal (XII) nerve can provide facial tone and symmetry at rest, and, in some cases, volitional movement with training. A temporary nerve transfer to these nerves is sometimes performed as a “babysitter” procedure while awaiting axonal growth through cross-facial nerve grafts.

When nerve repair or nerve transfers from the contralateral face or donor nerves are not feasible, such as after motor endplate degeneration has occurred in the facial muscles, innervated free muscle flap transfers can restore facial movement to the lower face. Muscles commonly used for facial reanimation include the gracilis, pectoralis minor, serratus anterior, and latissimus dorsi, because of their thinness, good excursion, and low donor site morbidity. In addition to a microvascular anastomosis, an epineural nerve repair is performed either to a cross-facial nerve graft or to a donor cranial nerve such as the masseteric nerve.

2017

How well did you know this?
1
Not at all
2
3
4
5
Perfectly
16
Q

A 45-year-old man is evaluated because of a 5-cm mass at the angle of the mandible. A CT scan shows an intraparotid mass. Ultrasound-guided fine-needle aspiration shows benign findings. Which of the following is the most appropriate next step in management?

A) Chemotherapy and superficial parotidectomy
B) Follow-up evaluation in 3 months
C) MRI
D) Superficial parotidectomy only
E) Total parotidectomy

A

The correct response is Option D.

Most parotid tumors are benign, but they can grow to a large size and produce significant symptoms of discomfort and distortion of anatomy. The differential diagnosis is aided by the use of imaging, CT, or MRI to confirm the location of the mass. Ultrasound-guided fine-needle aspiration is a useful next step in diagnosis of the majority (92%, Sharma, et al). MRI would be superfluous in the present case since imaging is sufficient. Benign neoplasms include pleomorphic adenoma, mucocele, branchial cleft cysts, and lymph nodes. Malignancies include adenocarcinoma.

2017

How well did you know this?
1
Not at all
2
3
4
5
Perfectly
17
Q

A 14-year-old girl is brought to the office because of a 1-month history of a painful, growing lesion in the hard palate with “electric-shock sensations” on palpation. Results of incisional biopsy show adenoid cystic carcinoma of the minor salivary glands. Which of the following is the most appropriate next step in management of this patient?

A) Chemotherapy
B) CT scan and MRI
C) Excision with 1-cm margins
D) Excision with 2-cm margins
E) Radiation therapy

A

The correct response is Option B.

Although minor salivary gland tumors are much less common than major salivary gland tumors, minor salivary gland tumors are much more likely to be malignant. Additionally, pediatric salivary cancers represent only about 5% of all salivary cancers, but are also more likely to be malignant (almost 50% were malignant in Armed Forces Institute of Pathology series of 168 pediatric salivary gland tumors). Finally, the palate is the most common source of minor salivary gland tumors, which are more likely to be malignant and higher stage when detected.

The clinical presentation of paresthesias of adenoid cystic carcinoma (ACC) suggests perineural invasion. One series (University of Maryland) of 243 minor salivary gland tumors found 78% of them were malignant, and of those malignant tumors, 15% were ACC. Given the perineural invasion symptoms, imaging, in particular MRI, can detect perineural invasion and help plan the degree of surgery.

In this scenario, clinical exam pointed to perineural invasion, which should be imaged to plan for surgery. Chemotherapy is not used in the treatment of this disease. Radiation therapy alone is not usually performed, as this is considered a surgical disease. However, it can be used as adjuvant therapy in addition to surgery. Excisional biopsy usually recommends 1- to 2-cm margins. Patients with high-stage, perineural invasion, lymphadenopathy, or other signs of extensive disease may receive surgery with adjuvant radiation therapy. Regardless, ordering a CT scan and MRI is a reasonable initial approach before surgical treatment.

2017

How well did you know this?
1
Not at all
2
3
4
5
Perfectly
18
Q

A 59-year-old man with tongue cancer undergoes a hemiglossectomy, neck dissection, and reconstruction with a radial forearm fasciocutaneous free flap. On postoperative day 10, he fails a swallowing study for all food consistencies. Postoperative radiation therapy is scheduled to begin in 2 weeks. What is the appropriate next step in management?

A) Laryngectomy for aspiration
B) Percutaneous endoscopic gastrostomy tube placement
C) Revision of the free flap
D) Tracheoesophageal puncture
E) Observation with delay of radiation therapy for 10 weeks

A

The correct response is Option B.

Most patients who undergo hemiglossectomy can expect reasonable speech and swallowing function when reconstructed with a thin, pliable free flap, such as the radial forearm fasciocutaneous free flap, that facilitates unrestricted residual tongue movement. Although his swallowing may likely improve as he recovers from surgery and tissue edema resolves, this patient will need a feeding tube to maintain his nutrition at this time. Additionally, it can be difficult for many patients who have undergone substantial tongue resections to meet their caloric needs even if they pass their initial swallowing study during radiation therapy and short-term feeding tube placement may be indicated.

Tracheoesophageal puncture with placement of a one-way valve speech prosthesis is used to restore speech function in patients who have received a laryngectomy and does not apply to this patient. Revision of the free flap is not indicated in the early postoperative period as it is unlikely to significantly improve swallowing and may delay adjuvant treatment. A laryngectomy for aspiration would only be indicated as a last resort in a patient with chronic, long-term aspiration of oral secretions resulting in recurrent pneumonia, most commonly following more extensive tongue resections, such as a total glossectomy, including removal of the tongue base. Postoperative radiation therapy should be administered within 4 to 6 weeks of surgery for maximal effectiveness and, therefore, delaying for 10 more weeks may adversely affect this patient’s survival.

2017

rm fasciocutaneous free flap, that facilitates unrestricted residual tongue movement. Although his swallowing may likely improve as he recovers from surgery and tissue edema resolves, this patient will need a feeding tube to maintain his nutrition at this time. Additionally, it can be difficult for many patients who have undergone substantial tongue resections to meet their caloric needs even if they pass their initial swallowing study during radiation therapy and short-term feeding tube placement may be indicated. Tracheoesophageal puncture with placement of a one-way valve speech prosthesis is used to restore speech function in patients who have received a laryngectomy and does not apply to this patient. Revision of the free flap is not indicated in the early postoperative period as it is unlikely to significantly improve swallowing and may delay adjuvant treatment. A laryngectomy for aspiration would only be indicated as a last resort in a patient with chronic, long-term aspiration of oral secretions resulting in recurrent pneumonia, most commonly following more extensive tongue resections, such as a total glossectomy, including removal of the tongue base. Postoperative radiation therapy should be administered within 4 to 6 weeks of surgery for maximal effectiveness and, therefore, delaying for 10 more weeks may adversely affect this patient’s survival. 2017

How well did you know this?
1
Not at all
2
3
4
5
Perfectly
19
Q

Which of the following findings is most likely in patients who undergo superficial parotidectomy for treatment of sialadenitis?

A) Frey syndrome at 3 months postoperatively
B) Hematoma after the first 24 hours postoperatively
C) Permanent postoperative facial nerve dysfunction
D) Salivary fistulae
E) Tinnitus

A

The correct response is Option D.

Frey syndrome occurs with injury and abnormal regeneration of the auriculotemporal nerve, but is a late complication (median time at presentation: 11 months). Postoperative facial nerve dysfunction can occur in up to 60% of patients, but the majority (90%) resolve without need for operative intervention. Tinnitus is not a recognized complication of superficial parotidectomy, and hematoma is an early complication (<24 hours). Patients with sialadenitis alone have increased risk for developing salivary fistulae.

2017

How well did you know this?
1
Not at all
2
3
4
5
Perfectly
20
Q

A 70-year-old man is referred for evaluation directly after undergoing excision of recurrent squamous cell carcinoma and Mohs micrographic surgery. A photograph is shown. Pathology shows some extension onto the parotid capsule. Clinical examination shows two enlarged postauricular lymph nodes. Which of the following is the most appropriate management of this patient?

A) Cervical facial flap plus radiation
B) Direct closure alone
C) Superficial parotidectomy and direct closure
D) Superficial parotidectomy, neck dissection, and coverage with a cheek rotation flap
E) Superficial parotidectomy, neck dissection, and direct closure

A

The correct response is Option D.

Direct closure is being used more commonly in large Mohs facial defects, especially in the elderly, as it minimizes the disruption of skin tension lines. The contraindication in this case to closure alone is the nodal involvement and the extension into the parotid capsule. Given the parotid capsule extension, adequate margins at a minimum will require a superficial parotidectomy. Radiation alone plus flap closure would have a higher recurrence rate because residual disease is left in the capsule. The nodal enlargement requires further evaluation and given the options the best combination is superficial parotidectomy, neck dissection, and a local rotation flap.

2017

How well did you know this?
1
Not at all
2
3
4
5
Perfectly
21
Q

A 65-year-old man comes to the office because of slow-growing, painless masses within each of the parotid glands. He has smoked 10 cigarettes daily for the past 25 years. Superficial parotidectomies are performed, and pathologic examination shows papillary cysts and mucoid fluid as well as nodules of lymphoid tissue in both tumor specimens. Which of the following is the most likely diagnosis?

A) Adenoid cystic carcinoma
B) Hemangioma
C) Pleomorphic adenoma
D) Squamous cell carcinoma
E) Warthin tumor

A

The correct response is Option E.

Salivary gland tumors are relatively rare and make up about 3 to 4% of all head and neck neoplasms. The majority of salivary gland tumors (approximately 80%) originate in the parotid gland. Approximately 80% of parotid gland tumors are benign. Facial paralysis may be associated with malignant tumors and are a sign of neural invasion.

Warthin tumor (papillary cystadenoma lymphomatosum) is the second most common tumor of the parotid gland and is benign. Warthin tumors predominantly occur in men of 50 to 70 years of age, most frequently smokers, and are the most common bilateral salivary gland tumors. The histologic appearance of this tumor is very characteristic and is characterized by papillary cysts and mucoid fluid as well as nodules of lymphoid tissue.

Pleomorphic adenoma, also known as benign mixed tumor, is the most common tumor of the parotid gland. This tumor is histologically characterized by epithelial and connective tissue elements, with stellate and spindle cells interspersed within a myxoid background.

Adenoid cystic carcinoma is the second-most common malignancy of the salivary glands after mucoepidermoid carcinoma and exhibits a propensity for perineural invasion. There are three histologic subtypes: cribriform, tubular, and solid. The cribriform pattern has a classic “swiss cheese” appearance with cells arranged in nests separated by round or oval spaces. The tubular pattern has a glandular architecture, while the solid (or basaloid) pattern has sheets of cells with little or no luminal spaces.

Hemangiomas are the most common salivary gland tumors found in children and usually involve the parotid gland. Histologically, the tumors are composed of capillaries lined by proliferative endothelial cells.

Squamous cell carcinoma is a malignant tumor that rarely involves the parotid gland, in comparison to the skin and aerodigestive tract. It is histologically identical to squamous cell cancers arising from other sites with epithelial cells that form sheets or compact masses, which invade adjacent connective tissue. Round nodules of keratinized squamous cells known as “keratinous pearls” are the hallmark of well-differentiated squamous cell carcinoma.

2017

How well did you know this?
1
Not at all
2
3
4
5
Perfectly
22
Q

A 64-year-old post-menopausal woman is referred for evaluation and treatment of mandibular osteonecrosis. Discontinuation of which of the following medications should be considered?

A) Alendronate
B) Calcitonin
C) Estrogen
D) Raloxifene
E) Teriparatide

A

The correct response is Option A.

Bisphosphonate-related osteonecrosis of the jaw is a condition found in patients who have received intravenous and oral forms of bisphosphonate therapy for various bone-related conditions such as osteoporosis. The patient may develop exposed, nonvital bone involving the maxillofacial structures. Osteonecrosis may occur following minor trauma with decreased capacity for bone healing due to the effects of bisphosphonate therapy. Treatment may involve antimicrobial rinses, systemic antibiotics, systemic or topical antifungals, and discontinuation of bisphosphonate therapy.

Raloxifene is in a class of drugs called estrogen agonists/antagonists that have been developed to provide the beneficial effects of estrogens without their potential disadvantages. It is neither an estrogen nor a hormone. Raloxifene used to be called a selective estrogen receptor modulator (SERM).

Calcitonin is a synthetic hormone for the treatment of osteoporosis. The naturally occurring hormone is involved in calcium regulation and bone metabolism.

Teriparatide, a type of parathyroid hormone, is approved for the treatment of osteoporosis in postmenopausal women and in men who are at high risk for fracture.

Estrogen therapy with or without progesterone is approved for the prevention of osteoporosis in postmenopausal women. Estrogen reduces bone loss, increases bone density in both the spine and hip, and reduces the risk for hip, spine, and other fractures in postmenopausal women.

2017

How well did you know this?
1
Not at all
2
3
4
5
Perfectly
23
Q

A 55-year-old man undergoes resection of locally destructive floor-of-mouth cancer including a tooth-bearing segment of the mandible. Immediate reconstruction is performed using a fibula flap. Dental restoration with implants is planned. Which of the following properties of the fibula flap is most likely to ensure the greatest initial stability of the dental implants?

A) Cancellous bone thickness
B) Cortical bone thickness
C) Periosteal thickness
D) Total bone thickness
E) Total bone width

A

The correct response is Option B.

When dental implants are placed in thicker cortical bone, better initial stability is achieved at the bone implant interface, reducing micromotion during the process of osseointegration. Cortical bone thickness has also been shown to correlate with the amount of torque required to remove a dental implant. No significant correlation has been shown between removal torque and total bone thickness. The presence of osteogenic cells in cancellous bone is responsible for a biologic response, although it is not as important in improving initial stability as thick cortical bone. Periosteal thickness is not relevant to dental restoration.

2017

How well did you know this?
1
Not at all
2
3
4
5
Perfectly
24
Q

A 62-year-old woman comes to the office because of squamous cell carcinoma of the tongue and floor of the mouth. Examination shows a 3 x 3-cm partial defect of the tongue and the floor of the mouth. The lesion will be resected and the defect reconstructed at the same time using a submental musculocutaneous flap. Exposure of the pedicle of this flap allows which of the following levels of nodes to be exposed and sampled?

A) Level I
B) Levels I and II
C) Levels I, II, and III
D) Levels I, II, III, and IV
E) Levels II and III

A

The correct response is Option A.

The submental flap is a potentially thin flap. Its pedicle is the submental artery, which arises off the facial artery. The pedicle is described as traveling between the submandibular gland and digastric muscle belly. It also dives deep and lies in between the mylohyoid and geniohyoid. Exposing the pedicle requires incision along the mental region, then in a transverse incision 1 to 2 cm below the mandibular body.

Level I nodes lie within the submental region with the mandibular body being the superior border and the hyoid bone being the inferior margin. They are further divided into 1a-submental, which lies anterior to the anterior belly of the digastric, and the 1b-submandibular, which lies posterior to the anterior belly of the digastric. Thus, exposure to the pedicle alone also exposes all the level I nodes.

Level II nodes are the upper jugular group, which are clustered around the upper third of the internal jugular vein. The superior border is the skull base, the hyoid is the inferior border, the anterior border is the anterior edge of the sternocleidomastoid, and the posterior edge is the posterior edge of the sternocleidomastoid.

Level III nodes are the middle third of the internal jugular, with the hyoid being the superior border, cricoid cartilage the inferior border, and the anterior posterior borders the anterior and posterior edges of the sternocleidomastoid, respectively.

Level IV nodes are the lower third of the internal jugular with the cricoid cartilage as the superior border and the clavicle as the inferior border. Again, the anterior border is the anterior edge of the sternocleidomastoid, and the posterior edge is the posterior edge of the sternocleidomastoid.

2017

How well did you know this?
1
Not at all
2
3
4
5
Perfectly
25
Q

A 62-year-old woman is evaluated for lower blepharoplasty. On examination, negative vector is noted. Postoperatively, this patient is at increased risk for which of the following conditions?

A) Dystopia
B) Ectropion
C) Enophthalmos
D) Lagophthalmos
E) Proptosis

A

The correct response is Option B.

The finding of a negative vector places the patient at an elevated risk for lower lid malposition and ectropion. The negative vector refers to the anatomic relationship on lateral view of the maximum projecting point of the globe and the maximum projecting point of the infraorbital malar prominence. If the globe projects less than the malar prominence, a negative vector exists. Conversely, if the malar prominence projects more than the globe, a positive vector exists.

The negative vector finding indicates potentially deficient globe and lid support based on skeletal anatomy. Such patients will often have minor scleral show or lateral lid lag. It is important to recognize these findings prior to blepharoplasty surgery in order to surgically address the risks of ectropion via primary lid suspension during the blepharoplasty. Occasionally, lower lid blepharoplasty may be avoided if a negative vector is present and other conditions such as dry eye exist. Other findings or conditions that are associated with postoperative ectropion and lower lid malposition are: orbicular weakness, anterior lamellar shortage, inferior eyelid/orbital volume deficit, and eyelid laxity. Excessive or prominent middle lamellar scarring can occur after surgery, which can also lead to lid malposition.

Lagophthalmos is the inability to lower the upper lid fully and is a negative consequence of upper blepharoplasty due to excessive tissue resection or fibrosis. Enophthalmos is interior retraction or displacement of the globe related to increased orbital volume. This is unrelated to lower blepharoplasty surgery. Proptosis is an external displacement of the globe giving the appearance that the eyeball is extruding from the obit. This is most often associated with Graves disease, head trauma, and increased intracranial pressure. It can also be due to a retrobulbar hematoma after blepharoplasty, which is a surgical emergency due to the risk of blindness.

Dystopia refers to malposition of the globe related to skeletal changes of the orbit. This would not be a result of blepharoplasty, but can occur after facial trauma or facial tumor resection.

2016

How well did you know this?
1
Not at all
2
3
4
5
Perfectly
26
Q

A 42-year-old man is evaluated because of a 2-cm mass in the anterior floor of the mouth. Examination shows no palpable masses in the neck. In addition to surgical tumor removal, which of the following is the most appropriate additional step in management?

A) Radiation only
B) Radical neck dissection
C) Radical neck dissection and radiation
D) Selective neck dissection
E) Observation

A

The correct response is Option D.

A 2-cm mass with a clinically negative neck, pT2cN0, merits a supraomohyoid neck dissection because of the high risk of occult spread in this zone. A supraomohyoid dissection removes the lymph nodes in zones I, II, III; a modified neck dissection covers I-IV.

A smaller tumor, pT1N0, can be managed with a “tight ‘wait and watch’” strategy.

A radical neck dissection includes the sternocleidomastoid muscle and is not necessary for a clinically negative neck. Adding radiation therapy to a reflex neurovascular dystrophy is also not necessary for an N0 neck.

2016

How well did you know this?
1
Not at all
2
3
4
5
Perfectly
27
Q

Which of the following is the primary treatment for keratocystic odontogenic tumor?

A) Curettage only
B) Enucleation and chemoablation
C) Enucleation only
D) Marginal mandibulectomy
E) Segmental mandibulectomy

A

The correct response is Option B.

The most common benign tumors of the jaw are ameloblastoma (37%) and keratocystic odontogenic tumor (KCOT) (14%). Ameloblastomas are slow growing, occur in the 4th to 5th decades of life, and arise from odontogenic epithelium. KCOTs are locally aggressive, occur earlier in life, and also arise from odontogenic epithelium.

Curettage or enucleation results in higher recurrence rates. Addition of Carnoy’s solution (absolute alcohol, chloroform, glacial acetic acid, and ferric chloride) to the enucleated site for 3 minutes addresses the most common issue of local recurrence.

Segmental or marginal resection of the mandible is reserved for recurrence after resection locally.

2016

How well did you know this?
1
Not at all
2
3
4
5
Perfectly
28
Q

In the panoramic x-ray study (Panorex) shown, which of the following is the most likely diagnosis of the bilateral expansile lesions?

A) Ameloblastoma
B) Central giant cell granulomas (CGCG)
C) Neurofibromas
D) Odontogenic keratocysts
E) Osteosarcoma

A

The correct response is Option C.

Bilateral expansile lesions of the inferior alveolar nerve canal is pathognomonic of neurofibroma. The lesions on the Panorex are both expansile and not locally destructive of bone, as is common in ameloblastoma and odontogenic keratocysts. Central giant granulomas are most often multilocular, with cortical rupture and root atrophy. Osteosarcoma always has cortical destruction.

How well did you know this?
1
Not at all
2
3
4
5
Perfectly
29
Q

A 55-year-old woman is evaluated because of a 2-year history of an enlarging right-sided facial mass. Examination shows a roughly 4-cm firm mass in the right parotid region and a firm lymph node in zone III of the ipsilateral neck. Imaging does not show metastatic disease. Fine-needle aspiration of the mass suggests high-grade mucoepidermoid carcinoma of the parotid gland. Which of the following is the most appropriate treatment in this patient?

A) Neoadjuvant chemoradiation and reassessment of tumor response before additional treatment
B) Superficial parotidectomy with ipsilateral cervical lymph node dissection
C) Superficial parotidectomy with ipsilateral cervical lymph node dissection and postoperative adjuvant radiotherapy
D) Total parotidectomy with ipsilateral cervical lymph node dissection
E) Total parotidectomy with ipsilateral cervical lymph node dissection and postoperative adjuvant radiotherapy

A

The correct response is Option E.

This patient has T2N1M0 (Stage II) high-grade mucoepidermoid carcinoma, and total parotidectomy with ipsilateral cervical dissection is the appropriate treatment. Because the patient is lymph node positive, both an ipsilateral cervical lymph node dissection and postoperative adjuvant radiotherapy are indicated.

A superficial parotidectomy would perhaps be indicated in a low-grade mucoepidermoid carcinoma, but in a high grade lesion, total parotidectomy is more appropriate. Ipsilateral cervical lymph node dissection and postoperative adjuvant radiotherapy would be indicated in this patient, however.

Although total parotidectomy and ipsilateral cervical lymph node dissection are appropriate, two factors make postoperative adjuvant radiotherapy an important component of treatment: the node positive status of the neck, and the high grade of the tumor.

Chemotherapy remains somewhat controversial in the treatment of mucoepidermoid carcinoma, and, to date, is not a part of standardized therapy.

2016

How well did you know this?
1
Not at all
2
3
4
5
Perfectly
30
Q

A 55-year-old man is evaluated because of right-sided jaw pain, speech problems, and oral dysphagia 5 years after successfully completing a chemoradiation protocol for base-of-tongue cancer. CT imaging shows no signs of recurrence or distant metastatic disease. A panoramic x-ray study (Panorex) is shown. Which of the following is the most appropriate management of this constellation of symptoms?

A) Debridement of mandible
B) Hyperbaric oxygen therapy
C) Open reduction and internal fixation
D) Osteocutaneous fibular free tissue transfer
E) Rib graft

A

The correct response is Option D.

It is important to note that any patient experiencing pain like this should be worked up for recurrent disease. Nevertheless, this question addresses two issues in head and neck reconstruction. First, most non-oral cavity head and neck cancers are now treated primarily with chemoradiation protocols. This leaves surgery for salvage or for dealing with the morbidities of the therapy. In this case, the patient is now experiencing bilateral osteoradionecrosis with a pathologic fracture on the right. Had this problem stopped with the left side, debridement and letting the mandible sway to the ipsilateral side would be a form of management, although it can be disfiguring.

Secondly, from a reconstructive standpoint, this is a challenging case; but advances in flaps and preoperative modeling allow for managing such situations in one procedure. Given how long the fibular free flap can be fashioned and the ability to remove large central pieces of bone, surgeons can create two osseous segments vascularized off the same pedicle. Since many of these patients have difficult necks from a recipient vessel standpoint, decreasing the number of needed anastomoses is beneficial. Two free flaps can be performed at the same time or sequentially, but this adds increasing morbidity and complexity to the situation. In this case, a single fibular free flap with plates designed using stereolithographic modeling was adequate to restore the patient’s ability to eat and articulate. It also improved his occlusion, and, with debridement, stopped his pain. Postreconstructive imaging is shown.

Now that the right side is involved, simple debridement only would leave the patient potentially an oral cripple. By definition, the bone is of poor quality and simple open reduction and internal fixation will not restore the patient’s function or promote proper healing. Hyperbaric oxygen is used for osteoradionecrosis, but numerous studies now question its benefit in craniofacial bone, and it would not address the pathologic fracture on the right. Hyperbaric oxygen was shown to positively influence the osteoradionecrotic tissues prior to and post-free tissue reconstruction. Finally, a free bone graft in this situation, even with additional soft tissue coverage, would not address the major issue of hypoperfusion to the affected bone.

2016

this question addresses two issues in head and neck reconstruction. First, most non-oral cavity head and neck cancers are now treated primarily with chemoradiation protocols. This leaves surgery for salvage or for dealing with the morbidities of the therapy. In this case, the patient is now experiencing bilateral osteoradionecrosis with a pathologic fracture on the right. Had this problem stopped with the left side, debridement and letting the mandible sway to the ipsilateral side would be a form of management, although it can be disfiguring. Secondly, from a reconstructive standpoint, this is a challenging case; but advances in flaps and preoperative modeling allow for managing such situations in one procedure. Given how long the fibular free flap can be fashioned and the ability to remove large central pieces of bone, surgeons can create two osseous segments vascularized off the same pedicle. Since many of these patients have difficult necks from a recipient vessel standpoint, decreasing the number of needed anastomoses is beneficial. Two free flaps can be performed at the same time or sequentially, but this adds increasing morbidity and complexity to the situation. In this case, a single fibular free flap with plates designed using stereolithographic modeling was adequate to restore the patient’s ability to eat and articulate. It also improved his occlusion, and, with debridement, stopped his pain. Postreconstructive imaging is shown. Now that the right side is involved, simple debridement only would leave the patient potentially an oral cripple. By definition, the bone is of poor quality and simple open reduction and internal fixation will not restore the patient’s function or promote proper healing. Hyperbaric oxygen is used for osteoradionecrosis, but numerous studies now question its benefit in craniofacial bone, and it would not address the pathologic fracture on the right. Hyperbaric oxygen was shown to positively influence the osteoradionecrotic tissues prior to and post-free tissue reconstruction. Finally, a free bone graft in this situation, even with additional soft tissue coverage, would not address the major issue of hypoperfusion to the affected bone. 2016

How well did you know this?
1
Not at all
2
3
4
5
Perfectly
31
Q

A 50-year-old man with a long-term history of sun exposure is evaluated because of a 2.5-cm biopsy-proven squamous cell carcinoma of the central lower lip. Physical examination shows no enlarged cervical lymph nodes. Which of the following is the most appropriate treatment for this cancer?

A) Combined surgery and radiation
B) Curettage and electrodessication
C) Photodynamic therapy
D) Surgical excision
E) Treatment with 5-fluorouracil for 3 months

A

The correct response is Option D.

Oral cavity cancers are staged based on the following criteria set forth by the American Joint Committee on Cancer (AJCC):

The cancer described in the scenario is a stage T2N0M0 cancer of the lip. Surgical excision with margin control is the most appropriate and probably the most common treatment for this lesion, although Mohs micrographic surgery may also be an option for surgical removal. Reconstruction would be performed after attaining adequate surgical margins and can be performed utilizing local tissue flaps in most patients.

5-Fluorouracil cream can be used in the treatment of actinic keratosis and superficial basal cell cancers. It is also used in the treatment of some squamous cell cancers, but its efficacy is lower and would not be indicated for a T2 cancer. In curettage and electrodessication, small tumors are scraped off with a curette and electrocautery destroys residual tumor and controls bleeding. It is used for superficial squamous cell cancers without high-risk characteristics in noncosmetically sensitive sites and would not be appropriate for this lesion. Also, surgical margins cannot be evaluated with this technique. Photodynamic therapy involves application of a photosensitizing agent, such as 5-aminolevulinic acid (5-ALA), that is taken up by the cancer cells. The next day, the medicated areas are activated by a strong light. The treatment selectively destroys cancer cells while causing minimal damage to the surrounding tissue. It is used to treat actinic keratosis and is not approved by the Food and Drug Administration for treatment of squamous cell carcinoma, although experience suggests that it is effective for small, superficial cancers. Radiation therapy alone can be used in cosmetically and functionally important sites such as the lip or in patients who are poor candidates for surgery because of medial comorbidities. Given the good functional and cosmetic prognosis of surgical resection and reconstruction in this patient, irradiation would likely be reserved for a recurrent cancer. Combined surgery and irradiation is usually reserved for T4-stage cancers or those with nodal metastases.

2016

How well did you know this?
1
Not at all
2
3
4
5
Perfectly
32
Q

A 58-year-old man is evaluated because of floor-of-mouth cancer that is invading the mandible. A segmental mandibulectomy and reconstruction with an osteocutaneous free flap that includes bone from the lateral border of the scapula are planned. The vascular pedicle supplying this flap is based on which of the following arteries?

A) Circumflex scapular
B) Dorsal scapular
C) Lateral thoracic
D) Thoracoacromial
E) Transverse cervical

A

The correct response is Option A.

The circumflex scapular artery, which is a branch of the subscapular artery, supplies blood to the lateral and medial borders of the scapular bone. The scapula free flap was first described in 1978 by Saijo. It can be harvested as part of a chimeric flap that includes other tissues supplied by the subscapular arterial system, such as the latissimus dorsi muscle, serratus anterior muscle, and scapular or parascapular skin. The tip of the scapula receives its blood supply from the angular branch of the thoracodorsal artery and has also been utilized as a pedicle for the inferior portion of the scapula. While the scapular bone is not as thick as the fibula bone, it provides adequate stability for mandibular reconstruction. Cutaneous branches of the circumflex scapular artery supply the scapular and parascapular skin and, therefore, a skin flap can be harvested simultaneously to close soft-tissue defects.

The transverse cervical artery and the dorsal scapular artery primarily supply the trapezius muscle and overlying skin. The thoracoacromial artery supplies the pectoralis major muscle and overlying skin. The lateral thoracic artery supplies both the lateral portion of the pectoralis major muscle and the skin in the axillary region.

2016

33
Q

A 25-year-old woman is evaluated because of facial swelling around the jaw and loosening teeth. The swelling has worsened progressively. Physical examination shows unilateral right facial swelling around the third molar. CT scan of the mandible shows a radiolucent, multicystic, unilocular lesion in the right mandibular angle and confirmed root resorption. Which of the following series of treatments is most appropriate for this patient?

A) Local curettage of the lesion followed by cancellous bone graft reconstruction
B) Neo-adjuvant radiation therapy, segmental mandibulectomy, and reconstruction
C) Segmental mandibulectomy and reconstruction
D) Segmental mandibulectomy, reconstruction, and postoperative chemotherapy after adjuvant therapy
E) Segmental mandibulectomy, reconstruction, dental rehabilitation, and sentinel node biopsy

A

The correct response is Option C.

The patient described has an ameloblastoma. Ameloblastomas are benign tumors of odontogenic origin. Treatment is surgical. Conservative management, such as local curettage, is associated with high recurrence rate. The most appropriate treatment is segmental mandibulectomy, reconstruction, and dental rehabilitation. Because ameloblastoma is benign, neither adjuvant therapy nor neoadjuvant therapy is indicated.

2016

34
Q

A 50-year-old woman with hypertension and diabetes comes to the office because of a large mid-palatal cancer. She undergoes a total palatectomy. A photograph is shown. Which of the following is the most appropriate method of reconstruction?

A) Anterolateral thigh free flap
B) Bilateral temporalis muscle flaps
C) Fibula osteocutaneous free flap
D) Iliac crest bone grafting
E) Rehabilitation with a palatal obturator

A

The correct response is Option C.

This patient has a bilateral maxillectomy defect following resection via a Le Fort I osteotomy. While small defects can be successfully addressed with prosthetic palatal obturators that fit through the wound margins and clasp to the remaining teeth, larger defects can rarely be obturated because of their weight and instability due to lack of dentition and an alveolar ridge. Bone grafting is also not an option for such a large defect, particularly one resulting from a malignancy where postoperative radiation therapy is likely. In general, bone grafts are only indicated in benign conditions with bone loss less than 5 cm in length. Even when these conditions are met, they require coverage with well-vascularized tissue. Temporalis muscle flaps can be used to reconstruct palatal defects, but in this case, the skeletal elements of the mid face have been removed. Temporalis muscle flaps alone will result in loss of mid face projection. Similarly, the anterolateral thigh free and rectus abdominis myocutaneous free flaps are frequently used to reconstruct posterior maxillary defects but, though bulkier than temporalis muscle flaps, will not maintain midfacial projection in this patient with loss of the entire palate and premaxilla.

The most appropriate reconstruction for this patient is the fibula osteocutaneous free flap. This flap will restore midfacial height, width, and projection. It has adequate bone stock for osseointegrated implant placement dental restoration as well. The skin paddle is used to close the palatal defect, separating the oral cavity from the nasal cavity.

Several other osteocutaneous free flaps, including the iliac crest and scapular osteocutaneous free flaps, have also been used for similar reconstructions.

2016

35
Q

A 60-year-old woman with a history of squamous cell carcinoma of the scalp treated with resection, skin grafting, and total scalp radiation therapy is evaluated for osteoradionecrosis of the cranial vertex. After full-thickness debridement, which of the following is the most appropriate method for soft-tissue reconstruction?

A) Coverage with a free latissimus dorsi musculocutaneous flap
B) Coverage with a pedicled trapezius musculocutaneous flap
C) Coverage with a scalp rotation flap
D) Negative pressure wound therapy
E) Skin grafting

A

The correct response is Option A.

In the patient described, with a history of previous radiation therapy and a full-thickness defect, a free latissimus dorsi musculocutaneous flap would be the best choice for soft-tissue reconstruction.

Skin grafting over an implant cranioplasty would not be a suitable option in this setting. Skin grafts could be considered in nonradiated, partial-thickness defects of the scalp or for coverage of scalp rotation flap donor sites.

A pedicled trapezius musculocutaneous flap would not reach the cranial vertex without undue tension and is better suited for full-thickness occipital defects.

Scalp rotation flaps are ideal for defects up to 8 cm in diameter in a nonradiated scalp. The patient’s history of previous radiation therapy would likely compromise the viability of a large rotation flap in this setting.

Negative pressure wound therapy over an implant cranioplasty in a radiated field would not allow for soft-tissue healing and coverage.

2016

36
Q

An otherwise healthy 59-year-old woman is scheduled to undergo resection of recurrent squamous cell cancer of the right temporal area 5 years after initial resection, superficial parotidectomy, limited upper cervical lymphadenectomy, skin grafting, and adjuvant radiation therapy. The anticipated defect will be 6 × 8 cm, involving the skin and subcutaneous tissues. The superficial temporal vessels cannot be identified. Which of the following is the most appropriate method of wound closure?

A) Lateral arm flap
B) Parascapular flap
C) Radial forearm free flap
D) Scalp rotation flap
E) Split-thickness skin grafting

A

The correct response is Option C.

The most appropriate method for wound closure is a radial forearm free flap, as it matches the thin skin and subcutaneous tissue of the temporal area and has a long pedicle that can reach recipient vessels in the neck. With a patient history of radiation therapy and superficial parotidectomy, the superficial temporal vessels are unlikely to be suitable recipient vessels. With a patient history of upper neck dissection, it is possible that suitable recipient vessels will only be found inferiorly, and thus, it is best to use a flap with a long, reliable pedicle. A split-thickness skin graft will likely fail in a radiated wound bed. A scalp rotation flap will likely include tissue in the radiation field, bring hair-bearing tissue into a non–hair-bearing area, and require a split-thickness skin graft for closure of the donor site. It could be considered in a patient who is not a candidate for free tissue transfer. The lateral arm flap and parascapular flap are slightly thicker than the radial forearm flap, but the principal reason to avoid these flaps in this patient is that their pedicle lengths are relatively short and may not reach the recipient vessels in the neck.

2015

37
Q

A 45-year-old woman is evaluated for a 2.5-cm, biopsy-proven squamous cell carcinoma of the left floor of the mouth. Imaging studies show cortical mandibular invasion, but no enlarged cervical lymph nodes or distant metastatic disease. A tracheostomy is performed. Immediate reconstruction is planned. Which of the following is the most appropriate surgical treatment for this cancer?

A) Wide local excision alone
B) Wide local excision and marginal mandibulectomy
C) Wide local excision and neck dissection
D) Wide local excision, marginal mandibulectomy, and neck dissection
E) Wide local excision, segmental mandibulectomy, and neck dissection

A

The correct response is Option E.

Oral cavity cancers are staged based on the following criteria set forth by the American Joint Committee on Cancer:

Primary tumor staging for oral cavity cancers (T)

TXPrimary tumor cannot be assessed

T0No evidence of primary tumor

TisCarcinoma in situ

T1Tumor = 2 cm in greatest dimension

T2Tumor >2 cm but not more than 4 cm in greatest dimension

T3Tumor >4 cm in greatest dimension

T4aModerately advanced, local disease

Lip – Tumor invades through cortical bone, inferior alveolar nerve, floor of mouth, or skin of face

Oral cavity – Tumor invades adjacent structures (e.g., through cortical bone, into deep extrinsic muscle of the tongue, maxillary sinus, or skin of face)

T4bVery advanced, local disease

Tumor invades masticator space, pterygoid plates, or skull base and/or encases internal carotid artery

The cancer described is a stage T4aN0M0 cancer of the floor of mouth. The T-stage is 4a based on cortical mandibular invasion demonstrated by radiographic imaging. Concern for mandibular invasion should be raised whenever a tumor abuts or is fixed to the mandible.

Mandibulectomy is indicated. Cortical invasion of the mandible is an indication for segmental mandibulectomy, in which the full thickness of the involved mandible and grossly disease-free margin are removed by osteotomies. Marginal mandibulectomy involves removal of the alveolar ridge and varying amounts of the inner or lingual table of the mandible depending on the location of the tumor. Marginal mandibulectomy is performed when cancers abut the mandible or invade the periosteum, but do not grossly invade the cortex of the bone.

Although clinically and radiographically the neck does not have nodal metastases (stage N0), surgical treatment is usually performed due to the risk for occult nodal metastases (20% or more in some studies). Such dissection also facilitates reconstruction by exposing potential recipient blood vessels for microvascular free tissue transfer.

2015

38
Q

In patients with ameloblastoma, which of the following cell populations gives rise to this tumor?

A) Cementoblast tissue
B) Enamel
C) Gingiva
D) Nerve root
E) Odontogenic epithelium

A

The correct response is Option E.

Ameloblastomas are benign odontogenic tumors derived from odontogenic epithelium. They are typically slow growing, and present in the fourth or fifth decade of life as a mandibular mass in most individuals (80%).

Odontogenic cementoblast tissue is not appropriate because this tissue gives rise to an extremely rare benign odontogenic tumor, the cementoma.

Gingiva is the mucosal covering of the alveolar bone.

Enamel is the dense compound of teeth.

2015

39
Q

Which of the following factors is most likely to increase the risk for osteoradionecrosis secondary to radiation therapy?

A) Dental caries
B) Edentulous mandible
C) Oral thrush
D) Osseointegrated implants
E) Radiation dose of 3500 cGy

A

The correct response is Option A.

Osteoradionecrosis (ORN) of the mandible is uncommon but can occur in up to 10% of patients after undergoing radiation therapy for oral cancers. The risk increases once radiation doses exceed 6500 cGy. Most reports of ORN have dental caries and extraction sites as precipitating factors. Periodontal disease can also lead to ORN. After undergoing radiation therapy, patients can develop oral candidiasis and xerostomia, and they may also have edentulous mandibles with dental implants after reconstruction. However, these do not increase the risk for ORN. Surgical resection and hyperbaric oxygen therapy are the mainstays of treatment.

2015

40
Q

A 68-year-old man with a history of laryngeal cancer treated with chemoradiation 2 years ago has a recurrence. He is scheduled for total laryngopharyngectomy with circumferential resection of the pharynx extending from the floor of the mouth to 2 cm above the manubrium. Which of the following is the most appropriate single-stage reconstruction?

A) Construction of a spit fistula
B) Coverage with an anterolateral thigh flap
C) Coverage with a deltopectoral flap
D) Coverage with a pectoralis flap
E) Use of gastric pull-up

A

The correct response is Option B.

The circumferential defect described in this patient requires coverage with a tubularized flap that can span the length of the defect and reestablish continuity of the alimentary track. Gastric pull-up is not a good option in this case because of its high morbidity and poor perfusion in the most proximal region of the gastric flap. Coverage with the pectoralis flap or deltopectoral flap is not an appropriate option because these flaps cannot be tubularized in a single-stage reconstruction. The spit fistula would not restore alimentary tract continuity, and it should only be used if no other reconstructive options are available or if the patient is medically unstable. The anterolateral thigh flap is the best option in this case because it can be tubularized to span the defect. In most cases, the resulting reconstruction is highly effective with restoration of swallowing function in the majority of patients.

2015

41
Q

An otherwise healthy 35-year-old woman is evaluated for a 3-cm left parotid mass. Physical examination shows weakness of facial muscles on the side of the tumor. CT scan shows several enlarged cervical lymph nodes. Parotidectomy is performed, and pathologic examination shows a mixed population of poorly differentiated epithelial cells and intermediate cells with occasional secretory cells and neural invasion. Which of the following is the most likely diagnosis?

A) Hemangioma
B) Mucoepidermoid carcinoma
C) Pleomorphic adenoma
D) Squamous cell carcinoma
E) Warthin tumor

A

The correct response is Option B.

Salivary gland tumors are relatively rare and make up about 3 to 4% of all head and neck neoplasms. Approximately 80% of salivary gland tumors originate in the parotid gland. Approximately 80% of parotid gland tumors are benign. Facial paralysis may be associated with malignant tumors and is a sign of neural invasion. Malignant tumors may also metastasize to the regional lymph nodes and to distant sites.

Mucoepidermoid carcinoma is the most common malignancy of the parotid gland and the second most common malignancy of the submandibular and minor salivary glands. Mucoepidermoid carcinomas contain two major elements: mucus-secreting cells, and epithelial cells of the epidermoid variety. Low-grade tumors are associated with a predominance of mucus-secreting cells lining cysts and intervening nests of well-differentiated epidermoid cells. High-grade tumors show few or no mucus-secreting cells and the epidermoid cells are poorly differentiated. Intermediate-grade tumors are defined by less cyst formation than low-grade tumors with nests of epidermoid and less differentiated intermediate cells. The biologic behavior of mucoepidermoid carcinomas correlates with their histologic grade. On the basis of the nerve invasion, the appropriate treatment for this patient includes radical parotidectomy with facial nerve sacrifice. A neck dissection should also be performed for high-grade lesions or those with suspicious adenopathy. Postoperative radiation therapy is usually recommended for higher-grade mucoepidermoid cancers.

Pleomorphic adenoma, also known as benign mixed tumor, is the most common benign tumor of the parotid gland. This tumor is histologically characterized by epithelial and connective tissue elements, with stellate and spindle cells interspersed with a myxoid background. Warthin tumor (papillary cystadenoma lymphomatosum) is the next most common tumor of the parotid gland and is also benign. Warthin tumors predominantly occur in males and are bilateral in 10% of patients. Histologically, they are characterized by papillary cysts and mucoid fluid as well as nodules of lymphoid tissue. Hemangiomas are the most common salivary gland tumors found in children, and usually involve the parotid gland. Like other hemangiomas, they are benign and characterized by a rapid growth phase around the age of 1 to 6 months, followed by gradual involution over 1 to 12 years. Histologically, the tumors are composed of capillaries lined by proliferative endothelial cells. Squamous cell carcinoma is a malignant tumor that rarely involves the parotid gland, in comparison with the skin and aerodigestive tract. When squamous cell cancers occur in the parotid gland, they are usually of metastatic origin, although primary squamous cancers of the salivary glands do occur. They are histologically identical to squamous cell cancers arising from other sites with epithelial cells that form sheets or compact masses that invade adjacent connective tissue. Round nodules of keratinized squamous cells, known as “keratinous pearls,” are the hallmark of well-differentiated squamous cell carcinoma.

2015

42
Q

A 47-year-old man is referred for examination of a composite defect of the mandible that extends from the right mandibular angle to the left mandibular angle. Which of the following flaps is most appropriate for reconstruction in this patient?

A) Fibula
B) Pectoralis
C) Radial forearm
D) Rectus
E) Scapula

A

The correct response is Option A.

The fibula flap is the most appropriate option in this case because a long section of bone requiring multiple osteotomies is needed. The fibula flap can provide 18 to 20 cm of bone and has both an endosteal and periosteal blood supply enabling shaping of the bone with multiple osteotomies. In addition, a skin paddle can be harvested with the flap to reconstruct the floor of mouth defect. The scapula and radial forearm flaps also provide bone and soft tissues; however, these flaps will not provide a long enough bone segment and cannot be reliably osteotomized in multiple locations. The rectus and pectoralis flaps are soft-tissue flaps, and their use in this case would result in marked deformity because the anterior arch has been resected.

2014

43
Q

A 55-year-old man is referred because of a 1-year history of ear and throat pain. Physical examination shows a 1-cm exophytic tumor of the anterior tonsillar pillar within the oropharynx. Biopsy of the tumor shows squamous cell carcinoma. Which of the following cervical lymphatic levels is most likely to be first involved in this patient?

A) I
B) II
C) III
D) IV
E) V

A

The correct response is Option B.

The anterior tonsillar pillar (palatoglossal arch) and tonsil are the most common site for primary neoplasms of the oropharynx. A 1-cm tumor (T1) at this location has a 71% incidence of cervical lymph node metastases. Oropharyngeal tumors arising at the base of the tongue have a similar incidence of lymphatic metastases, whereas oropharyngeal wall and soft palate T1 tumors only metastasize in 8 to 25% of cases. The most direct path of lymphatic drainage from the oropharynx is to level II (jugulodigastric) lymph nodes, which can be examined clinically. From level II the progression is sequential to levels III, IV, and V. It is rare to encounter a “skipped” level. The other less frequent lymphatic drainage pathways detectable only on imaging studies are to retropharyngeal and parapharyngeal nodes. Midline tumors can drain to bilateral lymphatic systems.

2014

44
Q

A 70-year-old man is evaluated following tumor resection. Physical examination shows a 4 × 4-cm defect of the right maxilla that includes all of the teeth posterior to the right canine (two premolars and three molars) but spares the right orbital floor. He did not undergo radiation therapy. He wishes to restore mastication, speech, and swallowing by the simplest means that will still be efficacious. Which of the following is the most appropriate method of reconstruction?

A) Fibula osteocutaneous free flap with osseointegrated implants
B) Osseointegrated implant–retained prosthesis
C) Prosthetic obturator
D) Rectus abdominis musculocutaneous free flap with a conventionally retained dental prosthesis
E) Temporalis muscle pedicled flap

A

The correct response is Option C.

Palatal obturators can adequately restore missing maxillary dentition as well as prevent oronasal leakage of air, liquids, and foods. They have the advantage of being removable, which permits visualization of the maxillary cavity for tumor surveillance. Prosthetic retention can be difficult or impossible in sizable defects, particularly when there are few teeth to stabilize the prosthesis. In this patient who has sufficient remaining maxillary teeth and the majority of the alveolar arch, the prosthesis is expected to have good stability, and would be the appropriate choice in a patient who wishes to avoid further invasive procedures.

The temporalis muscle flap can be transposed into the oral cavity and can be used for closing defects of the palate. However, this flap alone would not provide replacement of the missing dentition and is still more invasive than a palatal obturator. Additionally, the temporalis muscle flap results in marked temporal hollowing at the donor site. The rectus abdominis musculocutaneous free flap can close the palatal defect and restore shape to the cheek in patients with a unilateral maxillectomy. In combination with a dental prosthesis, the rectus abdominis musculocutaneous free flap can restore the patient’s appearance and function. However, the rectus abdominis musculocutaneous free flap is also an invasive procedure, and it can sometimes be challenging to inset the flap such that there is enough room in the mouth for a prosthesis. In a patient who has had a maxillectomy, there is generally inadequate remaining bone stock to place osseointegrated implants for prosthetic retention. The patient’s existing dentition should be adequate to support a prosthesis. The fibula osteocutaneous free flap and other osteocutaneous flaps can be used to close the palatal defect to prevent nasal regurgitation. The fibula osteocutaneous free flap can also accept osseointegrated implants for dental restoration due to the good quality of bone stock associated with this flap. However, fibula free flap and osseointegrated implant reconstruction is a very long and extensive procedure and can require more than one surgery to fully restore this patient, particularly if osseointegrated implants are not placed during the same procedure as the free flap reconstruction.

2014

45
Q

A 60-year-old man is evaluated for a 6-cm ameloblastoma of the right maxilla. Reconstruction using an osteocutaneous iliac crest free flap is planned. Which of the following arteries supplies arterial blood to this flap?

A) Deep circumflex iliac
B) Deep inferior epigastric
C) Descending genicular
D) Lateral circumflex femoral
E) Peroneal

A

The correct response is Option A.

The deep circumflex iliac artery is the major blood supply to the iliac crest free flap. It gives rise to periosteal branches and nutrient endosteal branches that supply the iliac crest bone posterior to the anterior superior iliac spine. It also gives rise to an ascending branch that supplies the internal oblique muscle and several musculocutaneous perforators that supply the overlying skin, allowing a myo-osseous or osteocutaneous free flap to be harvested, respectively. Use of the iliac crest osteocutaneous free flap has been described by several authors for maxillary as well as mandibular reconstruction, and the bone itself provides ample stock for accommodating osseointegrated implants for dental restoration.

The peroneal artery is the blood supply to the fibula free flap. Use of this flap is contraindicated when the peroneal artery contributes markedly to the blood supply of the distal lower extremity. The descending genicular artery is a branch of the superficial femoral artery and is the blood supply to the medial femoral condyle osseous free flap. Alternately, the medial superior genicular artery, another branch of the superficial femoral artery, can be used to supply this flap, but the pedicle is shorter. The descending branch of the lateral circumflex femoral artery is the blood supply to the anterolateral thigh free flap, which is a cutaneous perforator flap. The deep inferior epigastric artery is the blood supply to the rectus abdominis musculocutaneous free flap or the deep inferior epigastric perforator flap.

2014

46
Q

Which of the following types of head and neck tumors are most often associated with Epstein-Barr virus infection?

A) Larynx
B) Maxillary sinus
C) Nasopharynx
D) Tongue
E) Tonsil

A

The correct response is Option C.

Nasopharyngeal cancers are most often associated with Epstein-Barr virus (EBV) infections and arise from the mucous epithelium of the nasopharynx and are relatively rare in the United States. However, these tumors are endemic in Africa and East Asia, accounting for as many as 18% of head and neck cancers in China. Nasopharyngeal tumors are classified as either squamous cell cancers, keratinizing undifferentiated carcinoma, or non-keratinizing undifferentiated carcinoma. EBV infection is most strongly associated with the non-keratinizing undifferentiated subtype and is thought to increase malignant transformation. Nasopharyngeal cancers are most commonly treated with chemotherapy and radiation, with surgery reserved for recurrent or unusual cancers. Reconstruction of skull base defects is most commonly performed using microsurgical transfer of soft-tissue flaps. Alcohol and tobacco are the most common risk factors for head and neck cancers in general, and laryngeal cancers in particular, with cigarette smoking increasing the lifetime risk 5- to 25-fold. Other risk factors for head and neck cancers in general include cigar smoking, environmental exposures, dietary factors (red meat, betel nuts), and human papillomavirus (HPV) infection. HPV infections are most commonly associated with oropharyngeal cancers (tongue, tonsil). Significant risk factors for maxillary sinus cancers include cigarette smoking and environmental factors such as exposure to wood dust.

2014

47
Q

A 16-year-old boy comes to the office because of a progressive 6-month history of unilateral nasal obstruction and frequent epistaxis. Anterior rhinoscopy shows a soft, smooth, purplish lobulated mass filling the left nasal cavity. An attempted office biopsy results in profuse bleeding. Which of the following is the most likely diagnosis?

A) Dermoid cyst
B) Encephalocele
C) Hemangioma
D) Inverted papilloma
E) Nasopharyngeal angiofibroma

A

The correct response is Option E.

Nasopharyngeal angiofibromas, also known as juvenile nasopharyngeal angiofibromas, are benign but locally invasive vascular tumors that occur almost exclusively in adolescent males. Onset is most common in the second decade of life, and rarely occurs after age 25 years. Symptoms include unilateral or bilateral nasal obstruction, frequent epistaxis or blood-tinged nasal discharge, and conductive hearing loss from Eustachian-tube obstruction. In advanced stages, the angiofibroma can deform the nose, face, and orbits, as well as erode into the cranial cavity and put pressure on the optic chiasm, resulting in diplopia. Treatment is usually surgical with radiation and reserved for extensive cases such as those with intracranial extension. Preoperative embolization as well as hormone therapy with estrogens, may limit blood loss. Nasopharyngeal angiofibromas are highly vascular, and office biopsies should be avoided.

Inverted papilloma is a benign, locally aggressive neoplasm that arises in the nasal cavity and is associated with squamous cell carcinoma in approximately 5% of patients. The age of onset is usually between 40 and 60 years. Surgery is the primary treatment of inverted papilloma. Encephaloceles are neural tube defects that result in sac-like protrusions of the meninges (meningocele) or brain and meninges (meningoencephalocele) in various locations along the cranium, including intranasally. They tend to be bluish, soft, compressible masses that transilluminate. Biopsy may result in a cerebrospinal fluid leak. Hemangiomas are benign vascular lesions that are present at birth and characterized by a rapid growth phase around the age of 1 to 6 months followed by gradual involution over 1 to 12 years. A hemangioma would not be expected to first occur in adolescence. Dermoid cysts are derived from ectodermal and mesodermal tissue and may contain skin, hair follicles, sebaceous glands, and sweat glands. Dermoids are usually firm and noncompressible and most frequently occur as a slow-growing cystic mass over the dorsum of the nose, but may also be entirely intranasal. Dermoid cysts may also have a dural component and should not be biopsied until intracranial communication can be ruled out by x-ray studies. Encephaloceles, hemangiomas, and dermoid cysts are congenital nasal masses that occur in infancy rather than adolescence.

2014

48
Q

A 58-year-old man undergoes total laryngopharyngectomy for recurrent squamous cell carcinoma. The pedicle to the most appropriate flap for reconstruction of the resulting total circumferential pharyngectomy defect extending from the base of the tongue to the cervical esophagus is located between which of the following muscles?

A) Flexor carpi radialis and palmaris longus
B) Teres minor, teres major, and long head of the triceps
C) Teres minor, teres major, long head of the triceps, and humerus
D) Vastus lateralis and rectus femoris
E) Vastus medialis and rectus femoris

A

The correct response is Option D.

The best option for reconstruction in this patient requiring circumferential pharyngeal reconstruction is the anterolateral thigh flap. This fasciocutaneous flap is supplied by perforators from the descending branch of the lateral femoral circumflex vessels, which are a branch of the profunda femoris vessels. The descending branch runs between the vastus lateralis and rectus femoris muscles, not the vastus medialis and rectus femoris.

The radial forearm flap is based on the septum between the flexor carpi radialis and brachioradialis muscles in the arm. Although it can be used to reconstruct partial, noncircumferential pharyngectomy defects, it is not ideal for a long, circumferential defect in a previously radiated neck.

The pedicle runs between the flexor carpi radialis and brachioradialis, not the palmaris longus.

The circumflex scapular artery emerges from the triangular space in the back, which is defined by the teres minor, teres major, and the long head of the triceps. It is the pedicle to the parascapular and scapular flaps.

Option C defines the quadrangular space that transmits the axillary nerve and posterior humeral circumflex artery.

2014

49
Q

A 39-year-old woman undergoes a total parotidectomy with facial nerve preservation for mucoepidermoid carcinoma of the parotid gland. The final pathology report indicates microscopic disease at the deep margin, and follow-up imaging shows no gross residual disease. No detectable nodal oaA 39-year-old woman undergoes a total parotidectomy with facial nerve preservation for mucoepidermoid carcinoma of the parotid gland. The final pathology report indicates microscopic disease at the deep margin, and follow-up imaging shows no gross residual disease. No detectable nodal or other metastases are noted. Which of the following is the most appropriate next step in management?

A) Chemotherapy
B) Immunotherapy
C) Neck dissection
D) Radiation therapy
E) Reexcision of the deep margin

A

The correct response is Option D.

The patient described likely has a stage III tumor (T3 N0 M0). Standard management algorithms developed by the National Comprehensive Cancer Network recommend adjuvant radiation treatment when the persistence of positive margins relates to microscopic disease and not gross disease. If there is gross disease, either by physical examination or follow-up imaging, and it is resectable, then surgical resection of the residual disease should be done initially, followed by adjuvant radiation.

Chemotherapy for major salivary gland tumors is appropriate as a first-line therapy concomitant with radiation only in cases of squamous cell carcinoma. In patients with mucoepidermoid, adenoid cystic, and adenocarcinomas, the role of chemotherapy is mainly palliative and reserved for advanced situations of recurrent or distant systemic disease. The absence of standard chemotherapy protocols for these situations attests to the degree of response that can be expected.

Immunotherapy has no significant role in the treatment of major salivary gland malignancies.

Neck dissection is indicated for malignant salivary gland tumors with clinically positive nodes detected either on physical examination or with preoperative imaging workup. This applies to parotid tumors of either the superficial or the deep lobe. Typical imaging to identify nodal disease would include CT scan or MRI or both.

Neck dissections performed electively are rarely indicated, and only in very high-risk situations that are based on factors other than clinical and histologic features of the primary tumor. Radiation treatment is an effective treatment for negative necks with high risk of nodal disease, and is preferred over elective neck dissections.

Surgical resection of persistent disease is indicated when a previously treated parotid mass was incompletely resected, and the remaining tumor is gross and resectable, rather than just microscopic. If not resectable, then the patient should have definitive radiation treatment.

2014

50
Q

A 45-year-old woman comes to the office because of a painful 4-cm left parotid mass. Physical examination shows weakness of the left facial muscles. CT scan of the chest shows multiple lung nodules consistent with metastases. Parotidectomy is performed, and pathologic examination of the gland shows a cribriform (“Swiss cheese”) pattern of cells with perineural invasion. Which of the following is the most likely diagnosis?

A) Adenoid cystic carcinoma
B) Lymphangiosarcoma
C) Mucoepidermoid carcinoma
D) Pleomorphic adenoma
E) Warthin tumor

A

The correct response is Option A.

Salivary gland tumors are relatively rare and make up about 3 to 4% of all head and neck neoplasms. The majority of salivary gland tumors (approximately 80%) originate in the parotid gland. Approximately 80% of parotid gland tumors are benign. Malignant tumors are associated with facial paralysis and pain, although they may also be asymptomatic. Malignant tumors may also metastasize to the regional lymph nodes and to distant sites.

Pleomorphic adenoma, also known as benign mixed tumor, is the most common benign tumor of the parotid gland. This tumor is histologically characterized by epithelial and connective tissue elements, with stellate and spindle cells interspersed with a mixoid background.

Warthin tumor (papillary cystadenoma lymphomatosum) is the next most common tumor of the parotid gland and is also benign. Warthin tumors predominantly occur in males and are bilateral in 10% of patients. Histologically, they are characterized by papillary cysts and mucoid fluid as well as nodules of lymphoid tissue.

Mucoepidermoid carcinoma is the most common malignancy of the parotid gland and the second most common malignancy of the submandibular and minor salivary glands. Mucoepidermoid carcinomas contain two major elements: mucus-producing cells, and epithelial cells of the epidermoid variety. Low-grade tumors are associated with a predominance of mucus-secreting cells lining cysts and intervening nests of well-differentiated epidermoid cells. High-grade tumors show few or no mucus-producing cells and the epidermoid cells are poorly differentiated. Intermediate-grade tumors are defined by less cyst formation than low-grade tumors with nests of epidermoid and less differentiated intermediate cells. The biologic behavior of mucoepidermoid carcinomas correlates with their histologic grade.

Adenoid cystic carcinoma is the second most common tumor of the salivary glands and the most common malignant tumor of the submandibular, sublingual, and minor salivary glands. It is slightly more common in female patients and typically affects patients between the ages of 30 and 70 years with a peak incidence of 40 to 59 years. There are three histologic subtypes: cribriform, tubular, and solid. The cribriform pattern has a classic “Swiss cheese” appearance with cells arranged in nests separated by round or oval spaces. The tubular pattern has a glandular architecture, while the solid (or basaloid) pattern has sheets of cells with little or no luminal spaces. Adenoid cystic carcinoma usually exhibits a protracted course characterized by indolent growth and a propensity for perineural invasion, reported to occur in 20 to 80% of patients. Distant metastases, most frequently to the lung, are not uncommon.

Lymphangiosarcoma is a rare vascular tumor, which may be associated with prolonged lymphedema. These tumors are more commonly found in the extremities and under light microscopy appear as vascular channels with anaplastic endothelial cells.

2014

51
Q

A 40-year-old woman comes to the office because of a 5-year history of firm, painless swelling of the upper jaw that has increased progressively in size. CT scan is performed, and the lesion is shown. Resection is performed. Pathologic examination shows odontogenic epithelial islands bordered by palisading columnar cells. No invasion into the surrounding tissues is noted. Which of the following is the most likely diagnosis?

A) Ameloblastoma
B) Fibrous dysplasia
C) Nasopharyngeal angiofibroma
D) Osteosarcoma
E) Squamous cell carcinoma

A

The correct response is Option A.

Ameloblastomas are benign, locally invasive, odontogenic tumors accounting for 1% of tumors of the jaw and 10% of odontogenic tumors. Approximately 80% occur in the mandible and 20% occur in the maxilla. The peak incidence is in the third and fourth decades but may also arise in children and adolescents. Ameloblastomas may be radiographically found to be unilocular or, more commonly, multilocular with a “soap bubble” or “honeycomb” appearance. Treatment may be with enucleation and curettage or more radical resection. In rare cases, metastatic ameloblastoma and ameloblastic carcinomas have been reported.

Fibrous dysplasia is a benign hamartomatous lesion that has a diffuse, “ground-glass” appearance on x-ray studies. It is usually treated conservatively with shaving and re-contouring of the bone. Squamous cell carcinoma is usually associated with a painful mucosal lesion. Radiographically, bony invasion may be noted in locally advanced cases. Osteosarcomas are aggressive malignancies of mesenchymal origin that exhibit osteoblastic differentiation. They are the most common primary bony cancer. Their radiographic appearance is variable and may include nonspecific destruction of the bone similar to a carcinoma, mottled ossification, similar to fibrous dysplasia, but without well-defined borders, or lamellar ossification (sheets of neo-osteogenesis). Nasopharyngeal angiofibromas are benign but locally invasive vascular tumors that occur almost exclusively in male adolescents. Their symptoms include nasal obstruction but can eventually cause facial asymmetry and eye displacement, as they grow from the region of the sphenopalatine foramen first into the nasopharynx and choanae then into the paranasal sinuses, pterygopalatine and infratemporal fossae, orbit, and even the intracranial cavity. Radiographically, they are nonencapsulated, lobular soft-tissue masses that demonstrate intense uptake of intravenous contrast due to their highly vascular nature. Extensive bony destruction is usually not a feature, but bone may be remodeled or resorbed.

2014

52
Q

A 23-year-old man comes to the office because of a 1-year history of painless swelling, asymmetry, and loss of interdental relationships on the right side of the jaw. Physical examination shows crowding of the right-sided first and second molars and premolar dentition. The third molar has not erupted; in its place there is a palpable firm enlargement of the mandible. Panoramic x-ray study (Panorex) shows a 3-cm radiolucent unilocular cyst. Percutaneous biopsy of the cyst shows nonkeratinizing stratified squamous epithelium. Which of the following is the most likely diagnosis in this patient?

A) Dentigerous cyst
B) Gingival cyst
C) Gorlin cyst
D) Primordial cyst
E) Radicular cyst

A

The correct response is Option A.

The most appropriate answer is dentigerous cyst. This type of cyst develops in the context of an unerupted tooth, which can be seen below the cyst. The cyst is lined with benign nonkeratinizing epithelium and is caused by degeneration of the enamel reticulum of the unerupted tooth. It is the second most common type of jaw cyst. Two thirds occur in the mandible.

Gingival cysts appear most commonly on alveolar ridges of infants but can also rarely appear in adults. Their origin relates to rests of dental lamina, and, unlike dentigerous cysts, these contain keratin. Clinically, they are soft and fluctuant, and range in size between 1 and 15 mm.

Primordial cysts and odontogenic keratocysts are equivalent. They develop from rests of dental lamina and basal cell hamartomas. Therefore, unlike dentigerous cysts, these are lined with a keratinizing stratified squamous epithelium that is sometimes dysplastic. Their size range is 1 to 9 cm, and pain is a common symptom. Gorlin syndrome includes the association of multiple odontogenic keratocysts with multiple basal cell carcinomas, nasal deformity, skeletal abnormalities, calcification of the falx cerebri, and palmar or plantar pits.

Calcifying odontogenic (Gorlin) cysts are distinct from those above because they may be part cystic and part neoplastic. The histology shows features similar to the calcifying epithelioma of Malherbe (epithelium undergoing keratinization and calcification), and ameloblastic proliferations. Radiographically, they contain various amounts of radiopaque (calcified) material and are usually located anterior to the first molars ranging in size from 1 to 8 cm.

Radicular cysts are the most common type of jaw cysts and develop at the apex of a nonviable erupted tooth, from epithelial rests of Malassez in the periodontal ligament. As these cysts are inflammatory in nature rather than developmental, they are usually preceded by a periapical granuloma. Histology shows a fibrous shell lined with nonkeratinizing stratified squamous epithelium infiltrated with chronic inflammatory cells. Due to their painless quality, they most commonly occur as an incidental finding of routine x-ray studies of the maxilla. These cysts are radiolucent, and differ from dentigerous cysts because they are located at the apex of an erupted tooth rather than at the crown of an unerupted tooth.

2014

53
Q

A 51-year-old male carpenter requires a partial glossectomy for recurrent oral squamous cell carcinoma. He runs for five miles three days a week. He underwent radiation therapy two years previously. Microsurgical transfer of which of the following free flaps is most appropriate for reconstruction?

A) Deltopectoral flap
B) Rectus abdominis flap
C) Sural artery perforator flap
D) Vastus lateralis flap

A

The correct response is Option C.

Partial glossectomy defects require a small, thin, pliable flap for optimal reconstruction. The workhorse for glossectomy reconstruction has long been the radial forearm flap (RFF), which often requires a skin graft for donor site closure. However, various reports of donor site morbidity related to the RFF, including delayed healing, decreased grip and pinch strength, and radial nerve sensory problems, make this flap less suitable for a patient whose vocation involves manual labor. The sural artery perforator flap has become increasingly popular as an alternative donor site for very thin, pliable tissue. This flap, which usually arises from perforators from the medial sural artery, results in minimal donor morbidity. Muscle flaps are less desirable for intraoral reconstruction due to the need for an epithelialized surface and the difficulty in achieving skin graft adherence. The rectus abdominis flap is too bulky for partial glossectomy reconstruction. The deltopectoral flap, which arises from the 1st intercostal perforator, is rarely transferred as a free flap. Sacrifice of the vastus lateralis muscle in an avid runner is not recommended.

2019

54
Q

A 47-year-old Caucasian man comes to the office regarding a painful enlarging mass at the base of the tongue. He does not smoke cigarettes. The lesion measures 4.5 cm. A biopsy of the lesion is performed and shows (+) p16 staining, nonkeratinized squamous cell carcinoma. Further imaging and workup demonstrate an ipsilateral solitary lymph node measuring 2.3 cm. No distal metastatic disease is found. Which of the following best describes the stage of his disease?

A) Stage 1
B) Stage 2
C) Stage 3
D) Stage 4

A

The correct response is Option B.

The correct answer is Stage 2. Previously, this patient would have been Stage 3. The American Joint Committee on Cancer (AJCC) revised its staging system for squamous cell cancers that stain p16 positive. These lesions are related to the human papillomavirus (HPV) and have been found to be less virulent tongue base or oropharyngeal cancers. Recent studies have demonstrated that 5 year survival difference for patients with Stage 4 disease as <50% for HPV-negative patients and >70% for HPV-positive patients, thus prompting the AJCC to study and revise the staging system for HPV-positive oropharyngeal cancers. These lesions tend to be more sensitive to radiation therapy and chemoradiation and a better prognosis overall. Patients with HPV-related squamous cell cancers tend to be younger, male, and Caucasian. HPV-related squamous cell cancers now represent the majority of newly diagnosed oropharyngeal carcinomas in the United States. This new staging system for HPV (+) related cancers went into effect 1/1/2017.

2019

55
Q

A 44-year-old woman has a malignancy of the nasopharynx. She does not smoke cigarettes or drink alcohol. Which of the following viruses is the most common cause of this patient’s malignancy?

A) Epstein-Barr virus (EBV)
B) Herpes simplex virus (HSV)
C) Human immunodeficiency virus (HIV)
D) Human papillomavirus (HPV)
E) Human T-lymphotrophic virus-1 (HTLV-1)

A

The correct response is Option A.

There is ongoing investigation into the pathogenesis of infectious agents in cancer formation. Although increasing attention is focused on the relationship between HPV and head and neck cancers, HPV strains 16 and 18 are associated with the oropharynx, not nasopharynx location. In contrast, the Epstein-Barr virus (EBV) is associated specifically with nasopharyngeal cancers as well as Burkitt lymphoma. The other viruses listed are not routinely associated with head and neck cancers.

2019

56
Q

A 40-year-old woman undergoes surgery for a right parotid tumor observed to be invading the facial nerve. During surgery, a segment of facial nerve is resected, resulting in a 5-cm gap from the main trunk to the distal branches. Which of the following is the most appropriate treatment for this patient?

A) Cable nerve graft repair
B) Cross-facial nerve grafting
C) Masseteric nerve to facial nerve transfer
D) Reconstruction with an innervated gracilis muscle free flap
E) Temporalis tendon transfer

A

The correct response is Option A.

When a facial nerve has been divided or resected, the best outcomes for regaining function are usually obtained from direct repair or cable nerve grafting. Facial motor nerve recovery is observed even in cancer patients who have preoperative weakness and who receive postoperative radiation therapy.

When this is not possible, for example, when the nerve has been resected very proximally, up to the intracranial portion of the nerve, cross-facial nerve grafting between redundant branches of the normal contralateral nerve and the distal facial nerve stumps of the paralyzed side can produce excellent results with spontaneous symmetrical facial movement. In a patient with long-standing facial paralysis, cable nerve grafting or cross-facial nerve grafting will not be successful because the motor endplates of the distal facial nerve have degenerated. Performing a nerve transfer from a donor nerve, such as the masseteric (V), spinal accessory (XI), or hypoglossal (XII) nerves can provide facial tone and symmetry at rest, and, in some cases, volitional movement with training. When nerve repair or nerve transfers from the contralateral face or donor nerves are not feasible, innervated free muscle flap transfers can restore facial movement to the lower face. Muscles commonly used for facial reanimation include the gracilis, pectoralis minor, serratus anterior, and latissimus dorsi, due to their thinness, good excursion, and low donor site morbidity. In addition to a microvascular anastomosis, an epineural nerve repair is performed either to a cross-facial nerve graft or a donor cranial nerve such as the masseteric nerve. In patients who are not candidates for free muscle transfer or who refuse it, temporalis tendon slings can be used to suspend the face, usually at the oral commissure, to improve facial symmetry and function.

2019

57
Q

A 45-year-old man presents with a rapidly growing, painful mass of the left cheek. He has facial weakness on the left side. CT scan shows a left parotid tumor encasing the facial nerve; there are no abnormal lymph nodes. Which of the following is the most appropriate surgical treatment for this lesion?

A) Enucleation
B) Partial parotidectomy
C) Radical parotidectomy
D) Superficial parotidectomy
E) Total parotidectomy sparing the facial nerve

A

The correct response is Option C.

Although most (80%) of parotid tumors are benign, the parotid gland is the most common location for a salivary gland malignancy. Pain, paresthesia, and facial paralysis are signs of neural invasion and are usually associated with malignant tumors. Rapid growth, bony fixation, skin ulceration, and palpable nodal enlargement are also associated with malignancies.

Parotidectomy is removal of part or all of the parotid gland. While not true “lobes,” the facial nerve anatomically divides the parotid gland into superficial and deep portions. A superficial parotidectomy involves removal of the parotid gland superficial to the plane of the facial nerve and is appropriate for benign and malignant tumors confined to this portion of the gland. A less than complete superficial parotidectomy that still removes the entire tumor with a negative margin is referred to as a partial parotidectomy. A total parotidectomy involves removal of both the superficial and deep lobes. The facial nerve is carefully dissected and spared if it is not involved. Enucleation, which involves simple removal of the mass, is not indicated for malignant tumors and felt to be controversial for benign tumors, with many surgeons advocating against it. For those that consider enucleation, resection should include the capsule of the tumor, and it should be reserved for benign superficial tumors less than 4 cm in diameter.

A radical parotidectomy is indicated in this case of a malignant lesion invading the facial nerve. This procedure involves total parotidectomy with facial nerve sacrifice. If adjacent structures such as the skin, mandible, or temporal bone are involved, an extended radical parotidectomy may be indicated. In the clinically and radiographically node-negative neck, the decision to perform a neck dissection or treat the neck with adjuvant radiation versus observation usually depends on histologic factors and/or tumor subtype and grade.

2019

58
Q

A 79-year-old woman undergoes excision and elective neck dissection of a 2.5-cm invasive squamous cell carcinoma of the right lateral surface tongue. She has no history of head and neck cancer, and there is no radiologic or clinical evidence of nodal or metastatic disease. Wide excision with adequate margins and ipsilateral modified radical neck dissection is performed. Elective neck dissection is most likely to result in which of the following outcomes in this patient?

A) Decreased local recurrence
B) Fewer postoperative complications
C) Increased incidence of distant metastasis
D) Increased nodal relapse
E) Increased overall survival

A

The correct response is Option E.

This patient will have increased overall survival compared with a patient who does not have elective neck dissection. The patient described has Stage II (T2 N0 M0) oral cancer (larger than 2 cm but not larger than 4 cm, has not spread to lymph nodes with no metastatic disease). There has been much debate regarding management of the neck in patients with early-stage oral cancers. The two primary options include elective neck dissection (ie, at the time of the primary tumor resection) versus therapeutic neck dissection in the case of nodal relapse. In a prospective, randomized, controlled trial study of patients with T1 or T2 node-negative oral squamous cell carcinoma, patients received either elective neck dissection at the time of primary tumor resection or therapeutic neck dissection after nodal relapse. At 3 years, patients who underwent elective neck dissection had a higher rate of survival compared with the therapeutic surgery group (69.5 vs. 45.9%, P<0.001).

Patients who undergo elective neck dissection at the time of primary tumor resection have an increased number of postoperative complications and decreased nodal recurrence. Distant metastasis was the same between the two groups.

2019

59
Q

A 64-year old woman comes to the office because of a nonhealing radiated scalp wound. Medical history includes resection of invasive basal cell carcinoma of the scalp, reconstruction with a scalp rotation flap, and high-dose postoperative radiation therapy (60 Gy) 10 years ago. Clinical examination shows a full-thickness wound consisting of erythematous, ulcerated, and necrotic skin, and exposed, foul-smelling skull at the base of the wound. Which of the following is the most appropriate next step in management of this patient?

A) Biopsy of the wound
B) Craniectomy with free flap reconstruction
C) MRI
D) Resection of involved scalp with split-thickness skin grafting
E) Vacuum-assisted closure (VAC)

A

The correct response is Option A.

The first step in managing this patient is biopsy of the wound to rule out cancer recurrence. Although the diagnosis is most likely osteoradionecrosis of the skull, one would not proceed with the next steps of management until recurrence of cancer is ruled out. In this patient, the management sequence would include a biopsy to rule out cancer recurrence, followed by CT scan to delineate the extent of the skull involvement. MRI would not delineate the extent of the bony involvement.

The rates of osteoradionecrosis occurrence vary in the literature (from 1.8 to 37%). Although the rate and severity of osteoradionecrosis are most consistently associated with doses of radiation exceeding 50 Gy, there are reports of osteoradionecrosis in patients who received doses as low as 30 Gy.

This patient would require extensive craniectomy by a neurosurgeon to debride the wound of necrotic bone and, in most cases, reconstruction with free tissue transfer. Vacuum-assisted closure would not be a viable option for this patient, nor would resection of the scalp with split-thickness skin grafting.

2019

60
Q

A 17-year-old boy is diagnosed with an infected molar and scheduled for dental extraction. Before the day of surgery, he is brought to the emergency department because of drooling, protruding tongue, bilateral woody edema of the submandibular region, and tenderness of the neck. Which of the following is the most likely diagnosis?

A) Ludwig angina
B) Periapical abscess
C) Peritonsillar abscess
D) Primary mononucleosis
E) Reactive lymphadenopathy

A

The correct response is Option A.

Drooling, protruding tongue, and woody edema (non-fluctuant) of the submandibular region are classic signs for Ludwig angina, or deep space infection of the floor of the mouth. The source is frequently dental periapical abscess, often molar in origin where the mandible bone is thinner, allowing the infection to spread to the floor of the mouth. Treatment includes ICU monitoring of the airway for possible impending intubation, antibiotics, and surgical drainage (and in this case, extraction of the tooth as well).

Mononucleosis is not characterized by woody edema. Peritonsillar abscess is more frequently unilateral with lateral pharyngeal space symptoms without woody edema. Periapical abscess of the tooth involves localized symptoms without generalized bilateral submandibular symptoms, although this can eventually lead to Ludwig angina.

Reactive lymphadenopathy is the result of head and neck infection and does not result in the symptoms described in this scenario.

2021

61
Q

A 45-year-old woman with no history of smoking presents with a 1.4-cm squamous cell carcinoma of the left anterior mandibular gingiva and an upper cervical neck mass. Full-thickness cortical invasion of the tumor into the superior mandible as well as a single enlarged lymph node is observed on CT scan. In addition to neck dissection, which of the following is the most appropriate surgical treatment?

A) Marginal mandibulectomy
B) Radiation therapy
C) Segmental mandibulectomy
D) Transoral laser surgery
E) Wide local excision with periosteal stripping

A

The correct response is Option C.

The most appropriate surgical treatment for transcortical invasion of the mandible is a segmental mandibulectomy, in which a full-thickness portion of the mandible is excised. Marginal mandibulectomy is reserved for cases in which the cancer stops at the periosteum or does not penetrate full-thickness through the cortex of the mandible. Wide local excision with or without periosteal stripping, and transoral laser surgery are not adequate treatment for this lesion. Radiation therapy for this advanced stage cancer is also not adequate. Combined surgery followed by postoperative radiation therapy is indicated.

2021

62
Q

For total laryngopharygeal defects, which of the following is an advantage of performing reconstruction with the jejunal free flap instead of the anterolateral thigh free flap?

A) Better voice production with tracheoesophageal puncture prosthesis
B) Greater feeding-tube independence
C) Less donor site morbidity
D) Lower flap loss rates
E) More straightforward inset

A

The correct response is Option E.

Both the jejunal free flap and the anterolateral thigh (ALT) free flap have been used to restore continuity of the hypopharynx and cervical esophagus following laryngopharyngectomy and have shown superior functional results and lower complication rates that previously used pedicled flap techniques. The jejunal free flap has become less popular in many centers in recent years because the ALT free flap has lower donor site morbidity, a higher rate of feeding tube independence, and equivalent or lower flap loss rates. Additionally, voice production with a tracheoesophageal puncture (TEP) prosthesis is considered superior with the ALT. The TEP speech with the jejujunal flap is characterized as wet and cavernous and, therefore, more difficult to understand. As a pre-formed tube, the jejunal flap is more straightforward to inset. The surgeon only has to perform a superior (base of tongue) and inferior (esophagus) mucosal anastomosis, whereas the ALT is a rectangular skin flap that must be tubed. The additional (vertical) suture line theoretically raises the risk for fistula formation. In a comparative study, Yu et al found the fistula rate to be 8% for the ALT free flap and 3% for the jejunal free flap. While the jejunum has inherent peristaltic activity, some studies have suggested that this movement does not improve bolus transit, but instead contributes to regurgitation and dysmotility.

2021

63
Q

A 10-year-old boy is evaluated because of a 6-month history of a slow-growing mass of the left cheek that is soft, nonfixed, and measures 4 cm in diameter. There is no associated adenopathy. Fine-needle aspiration confirms the diagnosis of pleomorphic adenoma; it is anterior/outside of the parotid gland. After complete excision, the parents should be counseled that the recurrence rate is most likely which of the following?

A) 1%
B) 10%
C) 20%
D) 40%

A

The correct response is Option B.

Pleomorphic adenoma is an uncommon lesion, and when it does occur, it is usually in the parotid gland. It does occur in the submandibular, sublingual, and other minor salivary gland locations in decreasing order of frequency. Pleomorphic adenoma is a benign lesion, but the recurrence rate is 6 to 15%. Given the location, wide margins are ideal, but care must be taken to spare the branches of the facial nerve.

2021

64
Q

A 57-year-old man undergoes resection of a malignant lesion involving the mandible. He does not smoke cigarettes. Which of the following flaps is most appropriate to use for reconstruction of the resulting mandibular defect from ramus to ramus?

A) Cadaveric bone graft with scapular free flap
B) Fibular free flap
C) Lateral thigh flap with rib graft
D) Myocutaneous pectoralis major flap with reconstructive plate
E) Osteocutaneous radial forearm free flap

A

The correct response is Option B.

The most appropriate flap for a large defect from ramus to ramus is a fibular free flap. It allows a large segment of vascularized bone and a skin paddle to be used for reconstruction.

The osteocutaneous radial forearm free flap is good for limited osteocutaneous defects, and has less bone available for reconstructing the mandibular defect than the fibular flap. It is not the best choice in this instance, because the bone defect is much larger than this flap can reliably provide.

The myocutaneous pectoralis flap has been used historically in mandibular reconstruction; however, it is prone to break down over the reconstruction plate, and is a less ideal choice than a free fibular flap to reconstruct the described defect.

A scapular free flap requires repositioning the patient and would not provide adequate vascularized bone to reconstruct the described mandibular defect. It is not typically used, even with cadaveric bone, to reconstruct large mandibular defects.

The anterolateral thigh flap is also used in soft tissue reconstruction, but does not provide vascularized bone for reconstructing the large bony defect in this particular case, and non-vascularized rib graft would not be a good substitute for a large mandibular defect.

2021

65
Q

A 54-year-old man with a nodule in the soft palate undergoes evaluation with CT scan and biopsy. Results show a malignant oropharyngeal tumor. In order to stage the cancer and determine treatment, which of the following must be performed?

A) Dental evaluation via panoramic x-ray study (Panorex)
B) p16 status via immunohistochemistry
C) Perineural invasion
D) Smoking status via urine cotinine
E) Speech evaluation via video nasoendoscopy

A

The correct response is Option B.

Oropharyngeal cancer affects the base of the tongue, soft palate, tonsils, and posterior pharyngeal wall. The most recent National Comprehensive Cancer Network (NCCN) staging guidelines require HPV status to determine staging. HPV status is determined by p16 status via immunohistochemistry on the biopsy sample. The other answers are incorrect because perineural invasion is an important prognostic factor, but is not included in current staging criteria; smoking is related to cancer, but its status is not required for cancer staging; and dental evaluation and speech evaluation are clinically indicated if needed and are not for cancer staging.

2021

66
Q

Compared with oropharyngeal squamous cell carcinomas associated with tobacco and alcohol, which of the following is true about oropharyngeal squamous cell carcinomas associated with human papillomavirus (HPV)?

A) They are associated with active HPV infection in the partner
B) They are more resistant to radiation therapy
C) They have a better prognosis, stage for stage
D) They more frequently occur in the hypopharynx
E) They occur more frequently in women

A

The correct response is Option C.

An estimated 53,260 cases of head and neck squamous cell carcinomas (HNSCCs) were anticipated in 2020 in the United States. 70% of the cases (38,380) will be in men. More cases in both women and men will be associated with human papillomavirus (HPV), and it is anticipated that HNSCCs associated with HPV will soon outnumber cases of HPV-associated cervical cancer.

Interestingly enough, partners of patients with HPV-associated HNSCCs screened for the HPV16 subtype have the same occurrence rate as the general population (1.2 to 1.3%). HPV-associated HNSCCs are associated with a greater number of lifetime sexual partners (N ? 9) and a greater number of partners involved with orogenital sex (N ≥ 4).

HPV-associated HNSCCs typically occur in the oropharynx, whereas lesions on the larynx and hypopharynx are associated with the larger field affected by tobacco smoke and alcohol.

Stage for stage, the prognosis is better for HPV-associated HNSCCs. To prevent over-treatment (or to encourage less aggressive treatment), the eighth edition of the Cancer Staging Manual of the American Joint Committee on Cancer has revised the tumor, node, metastasis (TNM) classification of HNSCCs to differentiate between HPV-positive and HPV-negative tumors.

2021

67
Q

A 72-year-old man presents with a 2-cm, nontender, rapidly growing, violaceous intradermal papule of the cheek. Current medications include tacrolimus following renal transplantation. Biopsy of the lesion shows small, round, blue cells with large prominent nuclei that stain positive for cytokeratin 20. Which of the following is the most likely diagnosis?

A) Amelanotic melanoma
B) Basal cell carcinoma
C) Keratoacanthoma
D) Merkel cell carcinoma
E) Squamous cell carcinoma

A

The correct response is Option D.

Merkel cell carcinoma (MCC) is an uncommon and extremely aggressive cutaneous malignancy that is challenging to diagnose. Up to one half of patients will eventually develop a recurrence or a metastasis. There are approximately 2500 cases of MCC diagnosed per year in the United States. Eighty percent of MCCs are caused by the Merkel cell polyomavirus and the remaining 20% by extensive ultraviolet-mediated damage.

MCC is most common on sun-damaged areas, with half located on the head and neck and nearly 40% on the extremities. They usually present as nontender, rapidly growing, painless, single, red to violaceous, firm intradermal papules or nodules. The epidermis overlying the tumor is usually preserved, but ulceration or crusting is not uncommon. Their doubling time can be as short as 12 days. The clinical features of MCC are summarized in the acronym AEIOU: asymptomatic, expanding, immunosuppressed, older than 50 years, and ultraviolet-exposed fair skin. They are differentiated histologically by the small, round, blue cells that stain positive for cytokeratin 20.

The surgical treatment of MCC consists of wide local excision with one to two centimeter margins, inclusive of the underlying fascia. Management of regional disease is critical with this tumor. A clinically negative nodal basin will be evaluated with a sentinel node biopsy while a patient with a clinically positive nodal basin will be offered a therapeutic complete lymph node dissection. Postoperative radiation is offered to patients with tumors greater than 1 cm, close/positive margins, or nodal involvement.

Melanoma cells stain positive for S100 and HMB-45. Basal cell carcinomas stain positive for Ber-EP4. Squamous cell carcinomas stain positive for AE1/AE3.

2021

68
Q

A 77-year-old man is anticipated to have a 4-cm lateral segmental defect of the mandible after undergoing extirpation of an oral tumor. Reconstruction of the mandibular defect with an iliac crest bone graft is planned. Which of the following should be considered a CONTRAINDICATION to the use of nonvascularized bone graft for mandible reconstruction in this patient?

A) Defect greater than 3 cm
B) Defect involving the mandibular body
C) Extirpation of a malignant lesion
D) Need for adjuvant radiation therapy
E) Patient age greater than 75 years

A

The correct response is Option D.

In most cases, vascularized bone flaps provide a reconstruction superior to autologous bone grafts in the treatment of segmental mandibular defects. However, they require more operative time, a longer hospital stay, and specialized surgical teams, and they have the potential for greater donor morbidity. There is evidence that for many cases, bone grafts can be used with acceptable results. Patients undergoing radiotherapy for malignant tumors have a significantly lower success rate with bone grafting than those undergoing vascularized bone flaps. While associated with a need for radiation, a diagnosis of malignancy itself does not predict graft failure. Lateral defects have similar outcomes whether bone grafts or flaps are used, likely due to the lower torsional forces in these regions than at the symphysis. While there is no consensus on the optimal maximal graft length, many consider 6 cm to be the maximum advisable length for a bone graft in this context. Microsurgical free tissue transfer can be successfully accomplished in patients in their 70s and 80s, albeit with a higher rate of complications. Age should not be considered an absolute contraindication in this case.

2022

69
Q

A 64-year-old man with a T3N2 oropharyngeal tumor of the tongue and tonsillar fossa undergoes transoral robotic surgery (TORS). Which of the following is the most likely advantage of TORS over traditional treatments?

A) Patients undergoing TORS have less dysphagia
B) Patients undergoing TORS have a lower risk of hemorrhage than with open approaches
C) There is decreased incidence of postoperative infection
D) There is no need for reconstruction with a free flap
E) There is significant improvement in disease-free survival

A

The correct response is Option E.

Transoral robotic surgery (TORS) has been demonstrated to be comparable or advantageous when compared with nonsurgical treatment. Overall and disease-free survival and quality of life are comparable between TORS patients and patients who received nonsurgical treatment. TORS patients had significantly higher disease-free survival than patients who underwent open approaches in a meta-analysis. Risk of hemorrhage after TORS is 6%, whereas the risk is less than 2% in open approaches. Risk of dysphagia and infection are not significantly different from traditional treatment.

Functional outcomes are significantly less in TORS patients than in patients who received nonsurgical treatment. In one study, only 9% of TORS patients needed gastrostomy tube feeding 1 year post-procedure compared with 31% in the chemotherapy group.

Depending on the size of the defects, patients will need either locoregional or free flaps (radial forearm and anterolateral thigh flaps are the most common) after TORS. The incidence of needing free flaps is lower in TORS, but free flaps are not obsolete in TORS.

Contraindications to TORS include: tumor abuts carotid artery, resection will include greater than 50% of the tongue base or posterior pharyngeal wall, tumor is fixed to the prevertebral fascia, or midline tongue base tumors that can potentially cause injuries to both lingual arteries.

2022

70
Q

A 59-year-old man is referred to the office with a diagnosis of esophageal adenocarcinoma. After esophagectomy, the surgeon elects to perform a supercharged pedicled jejunal flap for complete esophageal reconstruction. Which of the following is the most likely recipient site complication?

A) Chylothorax
B) Fistula
C) Flap failure
D) Hematoma
E) Isolated neck cellulitis

A

The correct response is Option B.

All of the complications listed in the options have been described in relationship to esophageal reconstruction with a jejunal flap. The most common complication involves abdominal wound infection, occurring in 21% of patients. The most common medical complication is pneumonia, occurring in 18% of the patients.

Regarding the recipient site, in multiple reviews of esophageal reconstruction with jejunal flaps, fistulas have the highest complication rate (up to 14%). The rate of complication for neck cellulitis, hematoma, flap failure, and chylothorax are 5%, 4%, 1%, and 1%, respectively.

2022

71
Q

A 70-year-old man with a history of smoking and oromandibular cancer is scheduled to undergo extirpation, virtual planned fibular mandibular reconstruction, and postoperative radiation therapy. The patient strongly wants dental restoration. Which of the following best describes the earliest that dental implants can be safely placed?

A) Immediately
B) After radiation therapy
C) After failure of dentures
D) One year postoperaratively
E) Never

A

The correct response is Option A.

Patients undergoing oromandibular reconstruction have functional as well as oncologic challenges. Being able to restore dentition in a timely fashion allows for improvement in oral feeding and articulation. Given the poor prognosis for many patients and risk for infection and osteoradionecrosis, many in the field either delayed placement of osteointegrated implants or generally dissuaded use of them altogether. With better ability to plan virtual reconstruction and involvement of an organized multidisciplinary team including extirpative and reconstructive surgery, as well as oral surgery and radiation oncology involvement, immediate dental restoration is possible with a high success rate.

2022

72
Q

A tumor that has metastasized to the mandible in a 70-year-old man is most likely to have originated from which of the following anatomic sites?

A) Colon
B) Kidney
C) Lung
D) Testis

A

The correct response is Option C.

The mandible is an uncommon site for metastatic tumor spread and is usually evidence of more widespread disease. They may arise from numerous sources and usually present with the discovery of a new nodular mass with or without ulceration. Imaging studies demonstrate radiolucent/hypodense lesions. In male patients, the most common age range is the fifth to seventh decades. In men, the most common primary site is the lung; in women, it is the breast. Of note, the posterior mandible appears to be the most common bony site of involvement. The mean survival time is 8 months.

2022

73
Q

A 50-year-old man presents with a 3-cm ulcerated oropharyngeal mass that has appeared and gradually enlarged over the past 4 months. Physical examination is otherwise unremarkable. CT scans of the neck and chest and liver function testing are within normal limits. In addition to biopsy of the mass, which of the following tests is most appropriate for this patient?

A) CT angiography of the head and neck
B) P-16 protein expression test
C) Sentinel lymph node biopsy
D) Serum IgA levels for Epstein-Barr virus
E) Urine cotinine assay

A

The correct response is Option B.

This patient’s lesion is concerning for oropharyngeal squamous cell carcinoma (SCC). To establish a diagnosis and direct treatment, fine-needle aspiration biopsy of the mass should be the next step in this patient’s workup. In addition to histologic examination, the specimen should be tested for p16 expression, which is a marker for HPV-associated cancer. HPV can infect the oral cavity and pharynx and cause oropharyngeal and, more rarely, oral SCC. HPV-positivity has important prognostic value and is staged differently than non–HPV-positive SCC’s. HPV-associated oropharyngeal SCC has a relatively indolent course and a better prognosis than non-HPV-associated oropharyngeal SCC. Treatment for early stage disease may either be radiation with or without chemotherapy and/or immunotherapy or surgery, including transoral robotic surgery. Survival and recurrence rates are similar for the two treatment strategies. The HPV vaccine protects against the types of HPV that cause oropharyngeal cancers.

Epstein-Barr virus (EBV) is implicated in the development of nasopharyngeal carcinoma (as well as certain types of lymphoma), not oropharyngeal cancer. While not used for diagnosis, measurement of EBV DNA may be done before and after treatment to monitor response.

Oropharyngeal cancers are not especially vascular, unlike juvenile nasopharyngeal angioma, which occurs in the nasopharynx and nasal cavity, not the oropharynx, so CT angiography is not indicated.

Sentinel lymph node biopsies are used to assess for occult lymph node metastases in the clinically N0 neck. Their use in squamous cell cancer is controversial and would not be the next step in making the diagnosis.

Urine cotinine assay is a test for cotinine, a byproduct of nicotine used to evaluate exposure to tobacco. While tobacco is an important risk factor for head and neck cancer, its use is not helpful in making a diagnosis or directing treatment.

2022

74
Q

A 74-year-old man with type 2 diabetes mellitus develops squamous cell carcinoma (SCC) of the mandible. His history is significant for bilateral distal fibular fractures sustained 10 years ago from a motor vehicle collision, which were repaired with open reduction and internal fixation. He states that the ability to eat and chew will be important to him. Which of the following statements regarding mandibular reconstructive options is correct?

A) The bone quality of the scapular flap is superior to the iliac crest flap
B) Dental rehabilitation with implants is more predictable in osteocutaneous radial forearm flap than the iliac crest flap
C) Free fibular flaps have a higher rate of donor site morbidity than iliac crest flaps
D) A history of distal fibular fractures is not an absolute contraindication to the use of a free fibular flap
E) Iliac crest bone flaps have lower failure rates than osteocutaneous radial forearm flaps

A

The correct response is Option D.

A history of fibular fracture is not an absolute contraindication to using a free fibular flap. According to a systematic review, iliac crest flaps have the highest rate of failure when compared to all the other methods combined (fibular, radial, scapular). While not contraindicated, dental implants are less frequently placed in patients reconstructed with radial forearm bone flaps. More donor site morbidities have been reported for fibular than iliac crest flaps. Scapular flaps provide inferior bone quality to fibular and iliac crest flaps.

2022

75
Q

A 46-year-old woman with no history of trauma has had 1 month of headaches and soft tissue swelling over the frontal sinus and lateral frontal bone. Today she has fever, mental status changes, and increased swelling and redness of the forehead. CT scan shows a frontal epidural abscess and opacified frontal sinuses. Which of the following is the most likely diagnosis?

A) Exostosis
B) Fibrous dysplasia
C) Frontal osteoma
D) Infected encephalocele
E) Pott puffy tumor

A

The correct response is Option E.

Pott puffy tumor is currently rare due to availability of antibiotics. Pott puffy tumor is an osteomyelitis and abscess that results from chronic frontal sinusitis. The clinical presentation and CT scan findings are typical for the presentation of this tumor. Fibrous dysplasia is a bony overgrowth tumor that is not infectious in nature. Encephalocele is a congenital condition with a skull defect. Exostosis and frontal osteoma are benign overgrowths of the bone that occur slowly. They are not associated with mental status changes.

2022

76
Q

A 58-year-old man with a history of base-of-tongue cancer treated with radiation therapy and chemotherapy 3 years ago presents with right jaw pain, severe malocclusion (crossbite), and drainage from a wound in the right cheek. A photograph is shown. CT scan shows a pathologic fracture and bony destruction consistent with osteoradionecrosis of the mandible. A biopsy is negative for recurrent cancer. Which of the following is the most appropriate treatment?

A) Hyperbaric oxygen
B) Open reduction and internal fixation
C) Pentoxifylline and tocopherol
D) Segmental mandibulectomy and osteocutaneous free flap reconstruction
E) Sequestrectomy and treatment with antibiotic oral rinses

A

The correct response is Option D.

Osteoradionecrosis (ORN) is a complication caused by high-dose radiation therapy to the head and neck, most commonly affecting the mandible. ORN may be painful and can be associated with pathologic fracture, purulent infection, osteomyelitis, and fistula formation. Marx classified ORN into three stages: Stage I, exposed alveolar bone without pathologic fracture that is responsive to hyperbaric oxygen (HBO) therapy; Stage II, disease that does not respond to HBO and requires sequestrectomy and saucerization; and Stage III, full-thickness bone damage or pathologic fracture that usually requires complete resection and reconstruction. In this case, the patient has a pathologic fracture (Stage III) as well as a fistula, and segmental mandibulectomy and osteocutaneous free flap reconstruction are indicated. There is no role for open reduction and internal fixation in most cases of ORN because healing is not likely to occur. Pentoxyifylline and tocopherol (vitamin E) are more recently described treatments that have been shown to be effective in the treatment of early stage ORN.

2022

77
Q

A 3-year-old boy is brought by his parents for evaluation of a unilateral lesion that overlays the angle of his jaw and extends onto the neck. The boy’s parents state that the lesion has been present since birth and grown gradually, and they are concerned it may become more problematic as he ages. On examination, the lesion is easily noticeable, soft in nature, and translucent with a pen light. The patient is referred to interventional radiology for sclerotherapy. Use of which of the following sclerosing agents is most likely to put this patient at increased risk for skin necrosis?

A) Bleomycin
B) Doxycycline
C ) Ethanol
D ) OK-432
E) Sodium tetradecyl sulfate

A

The correct response is Option C.

All of the options listed are appropriate for sclerotherapy, but ethanol has the highest complication rate. Treatment of lymphatic malformations (LMs) is dependent on multiple factors including size, anatomic location, cosmesis, pain, cystic type, and recurrent infections. For small and microcystic lesions where the entire LM can be excised, surgical resection is a good option, but for large, macrocystic, or cosmetically sensitive areas, sclerotherapy is considered first-line due to superior efficacy and lower complication rates. Sclerotherapy is performed by first aspirating the cyst(s) and then infusing a sclerosing agent. Each agent works through different mechanisms, but the end result is induction of an inflammatory
process and collapse and scarring of the cyst walls, thus decreasing the size of the lesion. Complications, including skin necrosis and nerve injury, are highest with ethanol at 18% on systematic review, compared to below 6% for the other agents.

2023

78
Q

A 52-year-old man with left maxillary sinus cancer undergoes maxillectomy, including removal of the medial and inferior orbital walls. Reconstruction is performed with titanium mesh and an anterolateral thigh free flap. During resection and reconstruction, it is necessary to retract the globe to reach the posterior region of the orbit. Postoperatively, the patient reports loss of vision in the eye with only light and dark perception. Prior to surgery, the patient had normal (20/20) vision. The patient is most likely experiencing which of the following complications?

A) Enophthalmos
B) Extraocular muscle entrapment
C) Oculocardiac reflex
D) Traumatic optic neuropathy
E) Vertical orbital dystopia

A

The correct response is Option D.

This patient is most likely suffering from traumatic optic neuropathy secondary to excessive retraction of the globe and, consequently, injury to the optic nerve during surgery. In addition to prompt ophthalmologic consultation, radiographic imaging should be obtained to rule out a foreign body impinging on the optic nerve, such as the titanium mesh, or hematoma of the optic nerve sheath, which may be indications for a return to the operating room. Otherwise, management is controversial with limited data in the literature to date, suggesting that no treatment is necessarily superior to observation. Some authors support the use of high-dose corticosteroids.

Extraocular muscle entrapment may occur if any of the extraocular muscles become caught on an edge of the titanium mesh. Enophthalmos may occur if the reconstructed orbital cavity volume is larger than it was prior to surgery. Similarly, vertical orbital dystopia (hypoglobus), when the eye is at a lower vertical level than the contralateral side, may occur if the reconstructed orbital floor is more inferior than it was prior to surgery. The oculocardiac reflex is a reduction of the heart rate (bradycardia) resulting from direct pressure placed on the extraocular muscles, glove, or conjunctiva. This may occur during resection and/or reconstruction involving the orbit. It usually ceases when the inciting stimulus (eg, retraction of the globe) is removed. It can also be treated by administration of an intravenous anticholinergic drug (eg, atropine or glycopyrrolate). Once sinus rhythm is restored, it is usually safe to proceed with surgery.

None of the other options will result in vision loss.

2023

79
Q

A 53-year-old man is evaluated because of a mass in the oropharynx. Examination of a specimen obtained on biopsy of the mass shows squamous cell carcinoma. Which of the following pathologic findings is the strongest indicator of a favorable prognosis in this patient?

A) Close (less than 5 mm) resection margins
B) Extracapsular extension
C) Human papilloma virus positivity
D) Nodal metastases
E) Perineural invasion

A

The correct response is Option C.

Human papilloma virus (HPV)-associated cancers, which arise in the oropharynx, have a better prognosis than non-HPV-associated cancers. In fact, in the 8th edition of the American Joint Committee on Cancer (AJCC)/International Union Against Cancer (UICC) staging system, HPV-associated oropharyngeal cancers are given a separate staging classification from non-HPV-associated cancers. Because of the difference in prognosis, all oropharyngeal cancer specimens should be tested for p16 or HPV DNA or RNA, all of which indicate HPV positivity, for staging purposes. In recent years, rates of oropharyngeal cancers have risen sharply in the United States, and the majority of these cancers are associated with HPV infection. Almost all of these cancers are caused by HPV16, a subtype of the HPV virus.

The presence of neck nodal metastases is the most important prognostic factor for oral squamous cell carcinoma. If present, there is a 50% reduction in survival rates. Perineural invasion, extracapsular (extranodal) extension, and close or positive resection margins are other negative prognostic factors and are usually considered indications for adjuvant radiation therapy.

2023